You are on page 1of 175

ALL MARCH PAPERZ

medicine gyne surg radio optho .


COMPILED BY DR.JAVED AHMED CHANDIO..
REMEMBER ME IN UR PRAYERZ
..
remember this z just compilation of papers
so key is not 100% .. correct it where it seems wrong
.

1ST MARCH , 2017


MEDICINE & ALLIED MORNING SHIFT
COMPILED BY: RK
Q No. 1 Stagnant hypoxia is caused due to:
a. COPD
b. Polycythemia
c. Severe anemia

Q No. 2 Brodie's abscess


a. Pyogenic osteomyelitis
b. Pyogenic arthritis
c. Tuberculous osteomyelitis
d. Tuberculous arthritis

Q No. 3 Regarding Acetaminophen most likely is:


a. Antiflammatory properties
b. is uricosuric
c. resembles methadone or something like that
d. its therapeutic window is less than aspirin
e. antitussive

Q No. 4 .transport tubular maximum for proteins:


a. 10 per minute or something like that
b. 20
c. 30
d. 40
e. 50

Q No. 5 True hermaphrodite:


a. XXY
b. XY
c. XYY
d. XO

Q No. 6 Complication of diabetes


a. Dry gangrene
b. Gas gangrene
c. Wet gangrene
d. Rest of options I cant recall
Q No. 7 Policeman tested for drugs and opium and cannabinoids were found in urine. The
first psychological response would be:
a. Delusion of grandeur
b. Denial
c. Depressive mood
d. Rationalization

Q No. 8 Pineal gland tumor compression in 10 years old boy causes decreased secretion of
melatonin:
a. Delayed puberty
b. Early puberty
c. Increase serotonin

Q No. 9 Person is in prolonged coma


a. Nucleus cerelus
b. Periaquiductal
c. Thalamus

Q No. 10 Anti-manic effect of lithium takes how much time:


a. 10 days
b. 15days
c. 5 days
d. 21 days

Q No. 11 Wave of depolarization passes from endo to epicardium:


a. RR interval
b. QT interval
c. QRS interval
d. Ventricular activation time

Q No. 11 Death is caused by bilateral damage to which nerve:


a. Vagus
b. Hypoglossal
c. Trigeminal
d. Facial

Q No. 12 First line of defense against microbial pathogens:


a. Skin
b. Complement
c. Opsonin
d. Immunoglobulins

Q No. 13 In normal person decreased acid secretion causes:


a. Abnormal protein digestion
b. Increase gastric secretion
c. Inhibition of acid secretion
Q No. 14 Regarding SLE most appropriate is:
a. Anticentromere antibodies acting against antibodies
b. B and t cells acting against their immune or something
c. Remaining were alien option

Q No. 15 You have to give your house officer some blood to check arterial blood gases:
a. Plasma in citrated tube
b. Whole blood in a heparinized syringe
c. Serum in something

Q No. 16 There was longest scenario and irrelevant information with one word i.e. Mallory
body
a. Alcoholic hepatitis

Q No. 17 Person has sharp wound on side of neck blood is gushing out, the senior registrar
will press on which anterior tubercle:
b. 6
c. 7
d. 10
e. 12

Q No. 18 Purkinje fibers are fastest in transmitting impulse because:


a. They have wider diameter
b. They have large number of sodium channels
c. Intercalated junction

Q No. 19 Stroke volume increased but neither pulse pressure nor mean arterial pressure
increased. What will be the effect?
a. Pulse pressure raised and mean arterial pressure increases
b. Pulse pressure raised and mean arterial not raised
c. Coronary blood flow increases
d. No change

Q No. 20 A child has on/off bruises and epistaxis increase BT, normal PT and aPTT
a. Vwb
b. Itp
c. Hemophilia

Q No. 21 Characteristic of VWB:


a. Prolonged APTT
b. Increase BT
c. Increase PT

Q No. 22 What is the composition of normal saline


a. 0.9 gm in 10 ml distilled water
b. 9 gm in 1000 ml distilled water
c. 0.9gm in 100 ml distilled water
Q No. 23 Maximum left ventricular pressure in a young healthy individual
a. 25
b. 50
c. 80
d. 120

Q No. 24 A young athlete injures his ankle and develops echhymosis in ankle...he can stand
on his toes but its painful:
a. DVT
b. Achilles tendon damage
c. Plantaris tendon damage

Q No. 25 Cervical segment of spinal cord does not have:


a. Lateral horn
b. Ventral horn
c. Dorsal root

Q No. 26 What is not present at L4:


a. Lateral horn
b. Ventral horn
c. Dorsal horn

Q No. 27 What travels along with lateral corticospinal tract?


a. Rubrospinal tract
b. Vestibulospinal tract
c. Spinothalamic tract

Q No. 28 L4 pain radiates to


a. In front of knee
b. Lateral calf
c. Post calf or medial calf can't remember
d. Post thigh

Q No. 29 Sesamoid cartilage in body


a. ALA of nose
b. Ear
c. Larynx
d. Epiglottis

Q No. 30 Which of following bronchodilator effect?


a. Albuterol
b. Ipratropium
c. Ipratropium (2 times, yes)
d. Atenolol
Q No. 31 About fibrocartilage:
a. It contains an amorphous substance
b. It is present inside disc of joints
c. Once damaged cannot regenerate
d. Contains elastic fibers

Q No. 32 Regarding synthesis of aqueous humor:


a. Ultrafiltration
b. Ultrafiltration + active secretion
c. Diffusion
d. Active secretion

Q No. 33 Premalignant condition:


a. Behcet
b. Lichen planus
c. Pemphigus vulgaris
d. Pemphigoid

Q No. 34 Nonsmoker tobacco addict presents with growth in floor of mouth, diagnosed as sub
mucousal fibrosis:
a. Premalignant lesion
b. Dysplasia
c. Metaplasia
d. Benign tumor
e. Hyperplasia

Q No. 35 About sickness:


a. Feeling of unwell as perceived by patient
b. Perceived by physician
c. Clinical
d. Lab finding
e. Medically proven

Q No. 36 High blood flow, oxygen consumption of kidney will be:


a. Remains same
b. Decreased
c. High in medulla

Q No. 37 Na-K ATPase activity increased by:


a. Dopamine
b. Noradrenaline
c. Adrenaline
d. Insulin

Q No. 38 Most likely regarding acute tubular necrosis:


a. Hyperkalemia
b. Raised urea and normal creatinine
Q No. 39 Patient has bp 80/50, cardiac output of 2L/min, central venous pressure of 2 cm of
water, 30 lactate:
a. Cardiac tamponade
b. Congestive cardiac failure
c. Septicemic shock

Q No. 40 Tumor suppressor gene:


a. P53 for wilms tumor
b. WT1 for pancreatic ca
c. APC for colon cancer

Q No. 41 Nucleus disappears in which stage:


a. Late normoblast
b. Early normoblast
c. Reticulocyte
Q No. 42 Thyroid movement:
a. Prevertebral fascia
b. Pretracheal fascia

Q No. 43 Blood supply of thymus:


a. Inferior thyroid
b. Costocervical trunk
c. Thyrocervical trunk
d. Subclavian artery

Q No. 44 Isthmus crosses:


a. 2 to4 ring
b. Hyoid and thyroid
c. One muscle was also there

Q No. 45 Working women in kitchen cut radial artery accidentally, response will be:
a. B fibers
b. Post ganglionic C
c. Post ganglionic B
d. Delta
e. Alpha

Q No. 46 Regarding basophils, twisted question can`t recall options


Q No. 47 Right nasal and left temporal fibers gone:
a. Lesion at left tract
b. Nerve
c. Chiasm
d. Lesion at right tract

Q No. 48 Pancreas cytoplasm stains blue:


a. Lysosomes
b. Mitochondria
c. SER
d. PER

Q No. 50 With superior thyroid, which nerve lies


a. Recurrent laryngeal
b. External
c. Internal
d. Juglar

Q No. 51 S4 is produced:
a. Ventricular filling
b. Ventricular filing during atrial systole
c. Contraction
Q No. 52 Which of following has different life: (I was recalling my life lol)
a. Chondrocytes
b. Chondroblasts
c. Osteoblasts
d. Osteoclasts
e. Osteocytes

Q No. 53 Preganant woman taken only iron throughout her pregnancy, what will be
deficient?
a. Folic acid
b. Vitamin A
c. Vitamin B
d. Vitamin C

Q No 54 Chelosis, seborrheic, dermatitis, which vitamin deficiency?


a. Niacin
b. Riboflavin
c. Vitamin C

Q No. 55 Vitamin A deficiency:


a. Night blindness

Q No. 56 Seventeen dehyrogenase or something is absent, which of following conversion


reaction will be stopped:
a. Cholesterol to progesterone
b. Progesterone to prognolone
c. Alien stuff, check book for its deficiency

Q No. 57 Have to prescribe vitamins:


a. Eggs
b. Milk
c. Vegetables

Q No. 58 Pseudostratified characteristics:


a. Nuclei at the base
b. Cells at base

Q No. 59 6 week child normal at birth now presented with jaundice and bilirubin stained
diaper:
a. Bliary atresia
b. Gilbert.
c. Crigler najjar
d. Hemolytic anemia

Q No. 60 Structure present in renal column


a. Inter lobar artery
b. Micro calyx
c. Collecting ducts
Q No. 61 Pneumothorax:
a. Chest wall spring out, ling collapsed

Q No. 62 Tracheostomy, heat loss:


a. Evaporation
b. Conduction
c. Convection

Q No. 63 Hemoglobin binds to:


a. Heptaglobin
b. RBC

Q No. 64 Man living in desert, drinks a lot of water, maximum absorption from?
a. PCT
b. DCT
c. CD

Q No. 65 Synergistic effect of loop and thiazide?


a. Hypercalcemia
b. Hyperkalemia
c. Hypokalemia
d. Hyponatremia

Q No. 66 Peripheral nerves surrounded by


a. Epi
b. Peri

Q No. 67 Synapses absent in:


a. Ventral column
b. Dorsal Column
c. Lateral Column
d. Sympathetic chain
e. Dorsal root ganglion

Q No. 68 Right dissociation:


a. Fetal hb
b. Increase pH
c. Hyperthyroidism
d. Fall of temperature

Q No. 69 Pregnant lady, pallor spoon Shaped nails:


a. Low MCV, high MCH, MCHC
b. Low MCV, MCH and high MCHC

Q No. 70 Pregnant lady, remote area:


a. Hepatitis A
b. Hepatitis B
c. Hepatitis E
Q No. 71 Old lady difficulties in swallowing, bruises skin, difficulty in typing:
a. CRERST

Q No. 72 Intra hepatic protein anabolism, extra hepatic protein catabolism:


a. Insulin
b. Glucagon
c. GH
d. Thyroid
e. Costisol

Q No. 73 In graves disease:


a. Decrease TSH

POOR RECALL
Q No. 74 Intestinal histology 2 questions, poor recall
Q No. 76 Cough, Sore throat, low grade fever, enlarge cervical lymph node X ray chest show
hilar lymphadenopathy:
a. Tb
b. Sarcoidosi

Q No. 77 RTA, talking more but less sense:


a. Boraca
b. Wernickes

Q No. 78 Pupillary light reflex:


a. 2, 3
b. 2, 4
c. 3, 5

Q No. 79 Which causes max volume increase:


a. Hypersonic NACL
b. Isotonic NACL
c. Dextrose

Q No. 80 RTA, can see but can`t recognize:


a. Frontal
b. Parietal
c. Wernickes
d. Visual association area

Q No. 81 Heavy water used to measure:


a. ICF
b. TBW
c. ECF

Q No. 82 Infective endocarditis:


a. Strep viridans
b. Strep
c. Staph
Q No. 83 Abscess of some area:
a. Staph
b. Strept
Q No. 84 Oxygen toxicity in sea divers, cause of death:
a. Drowsiness
b. Vomiting
c. Seizures

Q No. 85 Severe diarrhea:


a. Metabolic acidosis normal anion gap
b. Metabolic acidosis low anion gap
c. Metabolic acidosis high anion gap

Q No. 86 Respiratory acidosis scenario:


a. Asthma
b. COPD

Q No. 87 Transverse diameter of rib cage:


a. Diaphragm
b. Bucket handle movement
c. Pump handle movement
d. Joint movement

Q No. 88 Sequence of events


a. Damage, Bacteremia, Thrombosis, Perforation

Q No. 89 Penicillin-prophylaxis SE:


a. Headache
b. Poor recall

Q No. 90 Dysphagia, endoscopy shows circular patch at mid esophagus biopsy shows
polygonal cells with pleomorphic changes:
a. H- pylori
b. Fiber diet
c. Alcohol

Q No. 91 Patient had hypocalcemia and hyperphosphatemia, injection of pth increased cAMP
in kidney, cause:
a. Pseudo hypoparathyroidism
b. Hyperparathyroidism after surgery
c. Vit d intoxication
d. Vitamin D deficiency

Q No. 92 A long scenario with giant platelets:


a. Bernard scalier syndrome

Q No. 93 Drug receptor interaction?


a. Heparin k sath kuchh tha
b. Antacid MGSO4
c. Atenolol, bradycardia
Q No. 94 methaydopa MOA in hypertension:
a. Blocks adrenergic in CNS
b. Directly act
c. Block beta receptors

Q No. 95 Zona glomerulosa secretion controlled by:


a. Angiotensin 2
b. Rebin
c. Sympathetic
d. K
e. Na

Q NO. 96 Ulcerative colitis, DOC:


Q NO. 97 Circadian rhythm bronchoconstriction maximum:
a. Morning
b. Noon
c. Night
d. Sleep

Q No. 98 H2 receptors decreases acid secretion via:


a. Both decrease h+ secretion and vagal
b. Vagal
c. H inhibition

Q No. 99 atypical lymphocytosis sore throat


a. Monospot
b. Montaux

Q No. 100 Cervical branch of facial supplies:


a. Sternocleidomastoid
b. Platysma
c. Genoglosis

Q No. 101 Mycobacterium laprae


a. Intracellular acid fast
b. Extracellular acid fast
c. No Acid fast
d. Poor recall

Q No. 102 Senario upper respiratory infection lymphocyte normal decrease:


a. A gama was not in options
b. Common variable deficiency something
c. IgA deficient

Q No. 103 Travellors diarhea doc:


a. Norfloxacin

Q No. 104 Basal ganglia:


Anatomy question
Q No. 105 CSF options cant recall
Q No. 106 Female, which one of following having least chances of Miscarriage at 17 26
weeks
Q No. 107 sickness defined as:
Q No. 108 present and absent of vomiting data will be:
a. Nominal
b. Ordinal

Q No 109 Surgeon doing open abdominal surgery, HO spills 1 L NS drip on patients pelvic
brim, fluid will go into?
a. Right left paracolic gutter
b. Spleen
c. Back
d. Can`t recall options

Q No. 110 Basilar artery gives:


a. Anterior
b. Posterior
c. Middle

Q NO. 111 B lymphoma associated with which virus:


a. CMV
b. EBV
c. HPV

Q NO. 112 Pap smear findings, nuclei large present. Advice to do pap smear after 6 months:
a. Metaplasia
b. Hyperplasia
c. Dysplasia
d. Pre malignant

Q No. 113 senario of achondroplasia


a. Dominant
b. Recessive

Q No. 114 senario of conginital hypothyroidism


Q No. 115 physiologic shunt will be present when V/Q is:
a. Infinite
b. Zero
c. Normal
d. One

Q No. 116 Propylthiouracil mechanism of action


No. 117 Female can't stand from sitting but can walk:
a. Gluteus maximus
b. Gluteus minimus

Q No. 118 Longest pre erythrocyte phase shizonts in:


a. P. malariea
b. P. Ovale
c. P. Falciparum

Q No. 119 Injury near ankle can stand but painful walking
a. Achilles rupture
b. Plantar rupture

Q No. 120 Young type I diabetic female with morning hyperglycemia, insulin regime:
a. Long acting insulin twice daily
b. Short acting twice daily
c. Intermediate twice daily
d. Regular before each meal
e. Short and intermediate twice daily

Q No. 121 Pap smear ca cervix scenario, causative agent:


a. HPV 16
b. HPV 8
c. CMV

Q NO. 122 Lesion on vulva


a. Condyloma accuminatum

Q No. 123 Man trying to scratch his back and can`t, muscle damaged?
b. Lattisimus dorsi
c. Pectoral is major
d. Subscapularis
e. Suprainspinatus
f. Serratus anterior

Q No. 124 Cells in retina, self-generating impulse:


a. Bipolar
b. Amacrine
c. Ganglion
d. Horizontal

Q No. 125 Which vein directly drain to SVC:


a. Azygous
b. Hemi azygous
Surgery 1st march eve
compiled by

DR JAVED AHMED CHANDIO


Tried hard . sorry if any mistake of recall bias

Special thanks to DR.MOHAMMAD SHOAIB LODRO


REMEMBER us IN UR PRAYERS
KEY IS is not final yet SO ANS WITH REFERENCES ARE
ACCEPTABLE

Q.1
micturition reflex
A.self regenrative
B.supplied by sacral
Q.2
min pressure in aorta Ans.isovol contraction
Q.3
right bronchopulmonry segnents Ans. 3 .2..5
Q..4
Pt presented wd raised skin thickening on cheeck which was
premalignant
Actinic keratosis ???
intradermal nevus
Q.5
Anemic hypoxia
Methamglobinemia
Cynide poisoning
Q.6
Multiple chest abscesses which s most common organism?
Klebsiela
StAph aureus
Streptococci
Q.7
pt pain on walking in calf and relievd by rest due to
popletial artry
Q.8
how to develop doctor patient mutual.
Dignity maintain
Q.9
hyalin cartilage
A.collagen fibers invisible
B. Visible
Q.10
Spinal cord ends at Ans.
L1
Q.11
sever chest pain. pt died. on autopsy disecting aorta. wht is
cause medial necrosis
Q.12 Muscle important in unlocking of knee joint Popliteus
Q..13 intrstitial fluid measured by
A. Mannitol D20 B. Evans blue inulin C. Evan heavy water
d. mannitol inulin ????
Q.14 caroted sinus receptors
A.act as chemoreceptrs B. stimulated by chang in BP
Q.15 Accessory rib Compression of subclavian vessels n
brachial plexus
Q.16 Erythrocytosis secondry to erythtopoiton...
A. Renal adenoma B. Transitional
Q.17 Most common fracture of clavicle lateral 1/3
Q.18 Tumor invading to major vessels Renal cell carcinoma
Q.19 Splenic vessel in which Ligament Lenorenal
Q.20 sphinctr urethrae is supplied by Internal pudendal
Q.21 a mass in mid line, biopsy done, normal thyroid tissue,
what will be seen on histology.. Ans Cuboidal epithilium...??
Q.22. appndicular artry is branch of
iliocolic
post cecal?????
Q.23 Massive hyperkalemia
A.heavy exercise B. Chronic dirrhea
Q.24 From foramen caecum to midline neck diverticulum
remant f which forms thyrosglossl cyst
A.thyroglossol diverticulum
B.thyroglossol duct
Q.25 Femoral artery
A.Medial to femoral nerve in femoral traingle ???
B.Leaves through adductor longus
C.lies posterior to midinguinal point
Q.26 Ulnar nerve Adductor pollicis
Q.27 osmotic diuretics act on
PCT DCT Collecting duct
Q.28 Projection fibres
A.Internal capsule B.Corpus callosum C.Fornix
Q.29 Cushing syndrome
Leukocytosis Lucocytosis Eosinopenia Nutropenia
q.30 .Tactile signals from finger tips
Merkel Meissner Ruffini Free nerve ending
Q.31
walking barefoot. feet on pointed thing. Pointed object
touches the foot..reflex is initiated by
a.pacinian b. miesner c. ruffins d. free nerv ending
Q.32.
A child with exchange transfusion After 10 days develop
palmar erythema spreads to rest of the body alt and bilirubin
raised and dirrhea
A.Graft versus host????
B.Delayed transfusion reaction
Q.33 Excessice adh produced by some tumor
Hypo osmotic overhydration Hyperosmotic over hydration
Q.34 Pain and warmth by C fibres A delta A beta B fibres
Q.35 Collagen fibres regularly arraged in Dermis Epidermis
Reticular layer
Q.36
Sodium A.Regualted by adh B.Primary active transport in pct
C.Primary active transport in loop of henle
Q.37 Anterior cardiac vein drains into
Coronary sinus Right atrium Great cardiac vein Small cardiac
vein
Q.38 at apex iv sulcs which vein starts
A. Great cardiac b. Small cardiac C. Coronary sinus
Q.39 Ambigous genitelia 46xx
Congenital adrenal hyperplasia Mixed gonadal
Q.40. Blood flow is inversely related to Resistance
Q.41 Energy expenditure in inspiration 5% 10% 15%
Q.42 chronic smoker . suffered from emphysema
a. inc alvelr spac inc artrl O2 b. dec alvlr space inc artrl O2 C.
Dec alvelr spac dec O2 q.
43
pH 7.52 Hco3 30 pco2 40 Ans. Uncompnsated metabolic
alkalosis
q.44 gastric lymphoma associated with h.pylori gastritis
Q.45 human placenta ?? iska kia tha ? I dont remembr options
Chorioallantois
Q.46 prolong use of steroid ostioprosis and pron to bone
fractur..
Q.47 So there was this question about chances of what is
more 5 years after transplant Lymphoproliferative There was
no skin option
Q.48 prosthetic valv. disch on medication. come in ER with
sever epistaxis hb 6 TLC ..... platlet 75000 cause of bleeding
trauma thrombocytopenia drug induced
Q.49 turner karyotyping XO XX XY
Q.50 most aggrasiv tumer melanoma
Q.51 breath sounds on post thorax sitting position till
8rib ???10th ?? 12
q.52 source of carbohydrate for diabetic pt.?????
whol wheat
bread
Rice
Q.53 vit D richest source ?
butter milk fish cord liver oil caps
Q.54 Pt deranged aptt normal
factr 7 factr 12 antithrombin
Q.55 pain epigastrium and back . and on rt shoulder what is
origin of pain gall bladder
Q.56 Edema caused by=
a.lymphadenitis B.dec interstitial osmotic
Q.57. IVC blockd blood above the origin of azygous will be
shunted to A. Portal vein B. Left gastric //????
Q.58 At what plasma glucose conc .. glucose appears in urine
a. 180 B. 200 C. 250 ans
Q.59... Malignancy A. Invasion ans B. Pleomorphism C. Inc N/c
ratio
Q.60 Choloneris sinesis = cholongio carcinoma
Q.61. Syphilis...genital sores
Q.62 Edema ..protein 6 g..defect in a.basement membrene
Q.63Council man bodies apoptosis
Q.64. Cola colord urine high grade fever chills A. Falciparum
Q.65 High grade fever chills For 4 days black color urine on
antimalarials A. G6pd B. Falciparum
Q.66 Tenia solium Uncooked meat
Q.67 Middle meningeal Foramen spinosum
Q.68 Structure post to ureter a.gentofemoral B. Iliac vessels
Q.69 Ataxia, diplopia and nystagmus, which artry block Pca
Basilar ans Vertebral
Q.70 At birth ossifiction center is present in lower end of
femur.
Q.71 Winged scapula ... Long thorac nerve
q.72 Attachment of radius with A. Lunate B. Trapezium C.
Hamate
Q.73 Lactotrops inhibited by A.Dopamine B. GNRH
Q.74 Gallblader differiated from intestine Muscularis mucosa
Q.75 Part of bone formed from seccondary ossification A.
Epiphysis ans B. Diaphysis C. Epiphyseal plate D. Metaphysis
Q.76 Hartman pouch.... Ans. Common site of stone
obstruction
Q.77 Which one is not involved in normal regulation of body
temp a. Hyperventilation ans B. Sweating
q.78. 6 hrs aftr MI . pt developd cyanosis bcz of poor
ventilation impropr
a.V/Q b. rt to lt shunting c.slow cardiac asim key
79. Guitarist with loss of sensation in lateral palmer aspect of
palm Ans. Median
80. Child with coronary artery anuerysm Ans. Kawasaki
81..Body ist line of defence against microbes Skin ans
Neutrophills
82. First line of defence in tissues A. Macrophages ans B.
Neutrophils
83. floor of sub occipital triangle .??????
. Occipital artery Post auricular arety Vertebral artery
84. Monitoring of oral anticoagulant Pt ans Aptt
85. Factor 8 produced by A. Hepatocytes B. Endotelial cells
ans
86. Bordie abcess a. Pyogenic osteomylitis B. Tb osteomyeltis
87. Child dyspnoec, pseudomembrane cause is Exotoxin of
dyptheria ans Fibrinous exudate Endotoxin.of deptheria
88. Hormone sensitive lipase inhibited by Cortisol Thyroxine
Insulin ans
89. liver resected in a 45 yrs femal. for regrowth of remaining
hepatocytes into full sized liver wht is req
TNF EGF PGE steroid hepatocyte growth factor
90. Ashtmatic patient , p/o pain killer Pethindine Diclofenac
Ketorolac
91. frst responc against tumor formation is apoptosis NK cells
macrophages
92. transplantation kidney 6 months renal functions
detriorated. but aftr immunosuppression kidny func becom
normal. what is cause of detrioration ? acute cell mediated
ans asim key acute humerol rej chronic rej toxicity of
cyclosporin
93. forarm supination lost radial n musculoskeltl radial n
ulnar ulnar n median median n ant introcus radial n ant
intrrocus
94. female with breast CA and axillary nodes enlargmnt. on
histopathology of node . no metastasis. cause of lymph node
enlagmnt ? Sinus histocytosis ans infection lymphadinitis
95.. rt testicular tumer lymph drains to ipsilatrl pelvic n
paraaortic b/l para aortic b/L pelvic ipsilat pelvic contralatl
aortic
96.. rt 12th nrv damag causes a.rt half paralysys with atrophy
of rt side Ans b. rt half paralysis with out atrophy c. lt side
paralysis with atrophy d. lt side paralysis without atrophy
97. tidl vol 350 ml dead space 100 mil RR 18 calculate alveolar
ventilation rate 4 L 4.5 L ans 4.8 l
98. Aqueos diffusion A. Phenobarbitone B. Diazepam
99.scnerio of paan Ans. Submucosal fibrosis
100. Disease vs non disease ans. case control
101. 2x2 Ans chi square
102. How to covince society education of basic health
103. Axillary sheath formd by Ans.prevertebral
104. bundle o hiss Rca ans Lad
105. umbilical cord A.2 Umbical arteries ans B. 2 umbilical
veins
106. Basilar with ICA by Ans. post comunicating
107.potent chemotact c5a ans
108. alcohal detoxified by Peroxisome SER RER
109. farmer round mas aspergilosis ans
110. Dye normal on one side Just little on other side near
bladder A. pelvic kidney B. Unilateral agenesis
111 Scnerio joint abdomen pain anemia Ans . HBSS
112 Erythroblastosis fetalis has B+ve Rx is Ans. B -VE
113. a women agglutinating with anti -A but hav agglutinins
anti b v..v.v. tough scnro was.. Ans . A+VE asim long
question
114. hetrochromatin A. inactive B. Synthesize rRNa
115 vita A deficiency night blindness
116. Rectum Ans. Post r sacral 345
117. Rectum also supplied by median sacral
118. Pulse pressure increased by a.maxium resistance in
arteries then aorta B. Inc compliance of veins
119. 2 year old boy Macrocytic A. Vit b 12 B. Intrinsic factor
antibodies
120. Thyroid Ans. enclosed in pretracheal
121. Tall man normal ejaculation but 6 azoospermia cause lie
in A. Sertoli cells
122.
Which f follwng detect Direction of sound A. Auditory cortex
B. Cochlea. C. Sup olivary D .medial geniculate
123. Endometrial hyperplasia cause was askd Ans. ESTRogen
prolongd
124. Bulbar urethra rupture urine will go ..(sup perineal not in
options) A .scrotum i did B. thigh C. ishciorectal fossa D. Anal
triangle E. Deep pouch
125. A person with occipital sever headace came to u .. kafi
lamba scnro tha then askd u prescribe some drug now he
came again with dyspnea. Drug was A. Propanalol I did
B .methyl dopa C .aur yad nai
126. Reversible change in size shape dyspolarity.. A. Dysplasia
ans B .metaplasia C. Anaplasia
127. squamous in cervix Ans.. metaplasia
128. Loss of opposition f thumb= median
129. Long scnro About patient cant hold bottles mixd
sensation at thumb .. loss of flexion at distal phalax of
thumb.. .. I did A. Median nerve B. Radial C. Ulnar
130. A young male has petechai pallor lethargy .. definitv
diagnostic test A. Cbc B. Selology C. Molecular studies?? D.
Bone marrow aspirat
131. Anaerobic abcess after abd surgery by Bacteroides fragilis
132. Mainly in abdominal and gyne surgerios a. Bacteroides
ans B. E.coli
133. Medullary thyroid A. Inc calcitonin i did B. Inc Ca inc
Phosph C. Yad nai
134. Medial longitudinal arch main support Ans. TALUS
135. After marginal branch Blockd affectd will be Ans. AV
node
136. Paraysmpathetic ka tha koi options sahi tarh yad nai May
be aise tha koi Ans. causes dec diamter f pupil
137. Adrenergic Ans. Inc Glycogenolysis
138. T8 propioception loss T9 temp pain loss No option of T8
hemisection Ans. Was then T11(bcz asim key) all options were
wrong 100%
139. Smooth muscle A. Cardiac is specializd smooth muscle B.
Slow spikes C. Inner circular outer longitudinal ( i did. All
options were looking right)
140. PNS is Ans. Spinal nerves plus autonomic ganglia
141. 1st meotic completes Ans. A.just before puberty ans B. At
3rd month
142. Body motor image at A. Premotor B. Primary motor ans.
143. Respiratory alkalosis
A. Salicyclate poisng B. High altitude i did
144. Ventricles are completelt depolarized at A. Qrs complex
B. St segment ans past papers
145. During shock most important A. Adrenosympth system
B. Cns ischeamic response ans
146. Child got anemia and Other brother has history of
transfusion Ans. Thalasemia
147. Anemia Alt ast were give almost normal ALp Slightly
raised A. Stones in gall bladder B. Yad nai
148.. Pressure maximally dissipated at A.Arteriol cappilarey
ans B.venular capillry
149. Sensitive for SLE A. ANA ans B. Anti D.Dna
150. Streptococcal pnumonia Benzathine penicillin ans
151.. Diabetic 50plus comes in coma Rx is Ans. Reg insulin
152.5th month of pregnancy drug of choice. A. PTU ans B.
methimazole
153 During quiet inp ant.post and transverse inc mainly by A.
Diaphram ans B. Ext intrrcosatl
154. 59 year known Hypertensive on medication came in
comatose condition cause is
A. Acute renal failure ans B. Chronic Ren fail C. Diuretics 157.
Patient on warfarin Moniterd by Ans. PT
155. Pudendal nerve Ans. On lateral side of pudendal canal
156. Post triangle injurd is Spinal accessory
157. Sa node Ans generate spontsly
158.psudomonas long scenario cause = endotoxin
159. hemolytic anemia associated with
a. unconjugated b. kernincterus
160. gall bladdr differentiated from colon
= muscularis mucosa
161. women hemorrhage which part affectd = pct
162. gFR best measure= inulin
163. sucralfate with cimetidine= reduced clearance of
sucralfate
164. approach to liver=ligament teres
165.commnly repeatd of myxoid degenrtn= MVP
166.protooncogene expression
167.siblings hav also bleeding problem bleeding from
umbilivcal stump best test
aPTT
168. 1st pass metabolism maxium= oral
169 . intrinsic factor releasd from = fundus f stomach
170. a boy has small upper lower limb normal trunk pattern
is = ans autosomal dominant
171. medullary thyroid ca =
a. inc level of calcitonin ANS
b. 24,25 OH cholecalciferol
172. adh acts on??????
a. collecting duct
b.cortical collecting duct (I did)
c. dct

..
The end remember me in ur prayers
Surgery paper 1st March 2017
by HINA JAVED
( this z same paper . But compiled by another person so I have added)

1.After liver segment resection regeneration occur via

A) EGF B) TNF c) hepatocyte growth factor.

2.Metstasis occurs ..lose of E-cadherine

3....Pregnant lady with hyperthyroidism drug of


choicepropiothyouracil

4.After sugery of an Aids ptnt OT sterilized by .2%grutaldehyde

5.ptnt with pseudomonas infection having septecimia due


to.endotoxin

6.impt response to shock .cushing response

7.ptnt with megaloblast cells which area of stomch


affectedgastric fundusss

8limbmansack endocarditis associate mitral vavle prolapsed

9.after cyctourethrography of a male there is no shadow on left side


and milde shadow at pelvis .pelvic kidney

10.compound of ca in bone .ca hydroxylapetite

11..differnce of muscle in upper esophagus muscle and lower


esophagus muscles .ca calmoduline complex

12 connection between basilar artery and internal carotid artery is


..posterior communicating artery.
13anoxic hypoxia .methheamoglobenimia

14.marker of thyroid carcinoma ..calcitonin

15.GFR measured by .inuline

16.child with edema the defect in kidney.at basement memebrane

17..after damaged to right hypoglossal nerve Aright toung


muscle damged with atrophy Bright toung musle damaged without
atrophy..

18gastric lymphoma is associated with ..H pylori

19A man feel pain after 100 yards walk, problem in ..popletial
artry

20 when exccesive ADH then there ll be hypososmotic over


hydration

21 after long term use steroids ..osteoprosis plus bone frctur.

22harmann pouch in Gall bladder ,,,,,,,, common site of gallstone


obstrutction.

23sensitive treatment of SLE ..ANA

25heterochromatine is ..inactive form of DNA

26change of stratified squmous at endocervix is ..Metplasia

27.reverssible change of shape ,size is ..Dysplasia

28.councilman body present on histology .Apoptosis

30potnt chemotactic factor ..C5a


31 healing intention secondary imp feature ..contrction of
myofibroblast

32.breast cancer lymphnode swelling presentHistiocytosis

33..factor III prsnt in .endothelium

34.a ptnt after MI undergone ventilator in ICU got


cyanosed..mismatched V/Q ratio

35..right coronary Artry after giving marginal branch supplyAV


node

362 atrium 2 ventricle aorta ..pulmonary trunk all made by


..Endocardial coushin.

37.a ptnt chronic reanl failure on atopsy had


hyperthophied.parathyroid galand

38..brodies abscess presnt in ..pyogenic osteomylites

39osmotic diuresis occurs .pct

40.diluting segment of nrphrone. Thick limb ????not remember

41.extracellular space volume find out by ..heavy water +inuline

42..a case study done on people with disease and without


disease..case control study

43..loss of pain below the level of 9 on contralatlral side


hemisection at level 8

44..end of spinal cord at level.L1


45..during lactation Ammnoreha is due to inhibition of Gonadotrophic
releasing harmone.

46...prolactine release is control by .Dopamine.

47..turner syndrome is ..XO

48.rectum is realted posteriorly to 3rd to 5th sacral nerves

49.pain at epigastrium and shoulder pain agrevated by fatty diet the


organ is Gall bladder

50.a ptnt is short stature but length of limbs is right ,this trait is
autosomal Dominant

51..Hyperkelemia is due to ..metabolic acidosis.

52a ptnt with ph 7.43,pco2 40mg and hco3 is 28.................non


compensated metabolic Alklosis

53a ptnt with dilated vessels of heart ..kawasaki disease

54aortic aneurysm and aotic dissection .due to medial


necrosis

55..erthropoiten level raised inadenoma of kidney

56.spread of carcinoma to vessels is ..clear cell carcinoma

57.after blockage of ivc below level of azaygos vein there is another


way of drainage thorough left gstric vien.
58.maxialleryy sinus drain into .middle meatus

59unlocking of knee joint is. Popletius


60sampling done of lympnode in posterior triangle the nerve
damaged is spinal part of accessory nerve

61..a mass lesion on face with raised edgesactin kertosis or


squmous cell carcinoma

62.aggresive tumor is .melanoma

63tissue innate immunity is by ..macrophages

64..1st line of defense is ..skin

65a child after 6 months ..transfution of blood has dirreha


rahse..graft verses host reaction or cytomegalovirus

66.question about calculation.minut ventilation something

67.at wrist bone make joint with radius bone lunate(sacafoid


not in options)

68.Na actively absorb at .PCT

69..osmotic diuresis occurs at.PCT

70intensive edema due to decrease hydrostatic pressure of


interstesial fluid..dnt knw exact answer

71..middle meningial artry passes throughspinosum

72..dirction of sounds assessed due to medial colliculus..lateral


coliculus..auditory cortexI did auditory cortex

73,.a child with hypochromic microcytic anemia..brother also same


disease wat test you would adviseHB elctropherosis

74..PT normal APTT is abnormal which factor deficintfactor 7


75a ptnt on oral anticouagulant which value monitor ..PT

76a ptnt with anemia pain crises having disease with ..Hbss

77.a farmer having round mass uperr part of lung..Aspergilosis

78a man drank alcohol which cell orgenlle detoxify it.SER

79warmth and pain pass thorugh .C fibers

80a female with breast cancer which gene assoisated with it


BARC 1 and Barca 2

81.commonly hospiatal acquired organism inafction.satph aureus

82..which harmone reduce gastric motility..CCK

83.ectoparacite .Lice

84cyctecercosis due to eating ..eggs of tenia solium

85..velocity of blood in vessels invesly propotional to


..resistenc

86.betttal nut chewing causes .submucosal fibrosis

87A ptnt with semen profile , azospermia,,,the problem lies in


..sertoli cell

88.plecenta is A..conctinction between cpactum and


spongiusumb..chorioallantois I did that

89a female with heavy bleeding , endmetrium hyperplasia due to


..estrogenic stimulation

90carcinoma of testes spread to paraaortic lymph nodes


91..smooth musles are .A specialesd type make heart musle B
mostly having inner and outer layrs

92.which of the following is not method of temprture regulation

..hyperventilation

93.collagen found in ..reticular layer

94.a ptnt with sphlis from wher we get sample.genital sore

95.carotid sinus react to ..sudden cahng in bp

96about sinus Node what is ture. It generate impulse and


rhthmycally

97..bunddle of his is supplied by RCA

98..about pundle vessel wat true is lies lateral wall of


ischeorectal fossa

99 A female has blood showing reaction with Anti sera Aand


reaction in Anti sera D, blood group is .A positive

100a new born child of mother B negative having erythroblastosis


fetalis which blood transfusion will b good B negative

101 .A new born with jauindci have to study upon his liver ..dye
should b given through his. liagmentum teres

102..histopathology sign of metastatsisinvasion

103.A ptnt with hb 7 ,retic count incresdbilirubin incease which of


the following disease is,.heamolytic anemia

104..chronic use of steroids cause leucopenia


105epithelium of thyroid follicle is .cuboidal epithelium

106axialllery sheath is formed by ..prevertabral facia

107. About thyroid which is true.. enclosed by pretracheal facia

108..pulmonary segments on right lung 3,2,5

109apendicular artry is branch ofA ilioceacl b..post caecal I


did

110.a ptnt had prosthatic vavalve is on drugs having


thrombocytopenia that is A Drug inced answer

b idiopathic..

111iron storage in body is in form of .ferritine

112..a ptnt is hypertensive on diuretic too has hyperkelimiathis is


due to diuretics

113..a pregnanat lady with obstruction jaundice the best test


GGT level

114.at quite reaspiration pulmonary system use energy of


totall..5%

115..epinephrine causes .glycogenlysis

116..at quite respiration lungs can be auscaltatted by steth till


.A 6th rib B 8th rib answer

117in emphysema a increase alveoli area and increase o2


tensionb increase alveoli area and decreas o2 tension I did that
118 A ptnt with diplopia dysartheria is due to ..basilar
thrombosis

119 same question in paper 2

120which drug has aquous diffusion.pheobarbitol

121.renal threshold for gluocose is 180 I did or 200 0r 250

122when compare 2 things with table .chie square test

123.mutual trust develop between ptnt and doctr ..rasssuranc about


disease

124to give information about disease ..dirct educate ptnt or by


piniior

125..lower quardrent of breast carcinoma not drained


intoinferior pherinc nodes

126..a ptnt is hypertensive treated hospital now complain dyspnea.


Whivh drug cause this propranolol

127congenital cataract ..rubelllla

128urinary bladder which is true self generative

129..which of the following is true about rectum..supply by middle


rectal artry

130..which of the following presnt at time of birth.lower end


of femur

131..seconadry ossifaction cntre occurs at A epiphysis b epiphysial


plate
132blood pressure changes at .arteriole capilleri level

133which diet is best as carbohydrate for diabetic ptnt .a milk


b wheat

134..vit D richest source is cord liver oli

135a man cannot ooppos his finger aND thumb defct in


median nerve

136.weaknessss of thenar .median nerve..

137..palmar cutaneous lateral side suplly by ..median nerve

138.which of following not supply by ulnar nerve.

Aduuctor polices

1391st sign of vit A deficiency A xerophthelmiab keromacla..c


night blindness.. answer

140..calviavle fracture most common sit A lateral 1.3rd and 2,3rd


area B medial 13rd and laeral 2, 3rd

141medila arach key stone .Talus

142spinchter urethera supply by pudenadal nerv

143gynecological infction ..bateriod

144.pyogenic pernotitis baccteriod

145a man with hig grad fever jaundiceretc count icreas .black
water fever

146.a man with fever ,diiness,,chills .retc count


increase..falciparum malaria
147bulbar facia rupture urin leak into ..socrotum

148lipid accumulation occurs ..insuline or cartisole

149minimum pressur at aorta ..isovolumatric contraction

150cancer is due to .protoncogene progression

2nd march medicine evening 2017 compiled by dr Maham


unar
1. Following is Ketone bodies A. Acetoacetate B. Acetyl co a
2. A man had hyperExtension injury at cervical CORD leads to a. upper motor
neuron lesion in upper limb b. LMN lesion in lower limb c. LMN lesion in
upper limb d. loss of pain in lower limb e. loss of temp in lower limb
3. A man non smoker has slight cough for a week on x ray there is subpleural
coin lesion of 2 cm in right upper lobe ..diagnosis a. granuloma b. silicosis c.
squamous cell CA d. small cell Ca ans D
4. Stress related glycogenolysis in liver by? A.glucagon b. Cortisol c. Nore
epinepherine d. Epinephrine e. Insulin ans B
5. Insensible water loss a. 200 400 b. 400 600 c. 600 800 d. less than 200
6. A new born baby with tetany ca level given, on examination there is
murmur baby is suffering from infection by pneumocyst carini aspergillis
herpes simplex .diagnosis? A. Adenosine deaminase deficiency b. Digeorge
22q deletion c. b cell /plasma cell defect
7. A woman had MI 48 hour later she collapsed. On ECG there is irregular
low voltage waves.. pathophysiology? A. Dec CO B. Dec EF C. Inc refractory
period D. Inc speed of conduction of impulse E. Re entry current
8. Amyloidosis most commonly involve a. Brain b. Kidney c. Lungs d. skin
2
9. Common effect of nitroglycerine ,histamine isoprenaline theophyline a.
tachycardia b. Throbbing headache (not confirm) c. Dec force of contraction
in heart d. Postural hypotension
10. 20 year asthmatic finding ....fev1 75
11. A person had traumatic incident of upper limb follow 3 months he feels
pain and tenderness.. a swelling develops at the region.. on x ray swelling is
bright.. after some time it resolved.. mechanism? A. Metaplasia b. Dysplasia
c. Hypertrophy d. hyperplasia e. apoptosis
12. A 54 year woman has diplopia and weakness. On CT there is swelling in
ant mediastinum.. which of following confirm diagnosis a. Anti acetylcholine
receptor antibody
13. Research is done on function of thalamus MRI shows different sensory
stimuli.. which of following will produce minimal neurological activity in
thalamus? A. Visual b. Olfactory c. Auditory d. Touch e. Vibration
14. farmer's lung is due to? A. Coal dust b. Cotton fiber dust c. grain dust d.
asbestos
15. apex of lung a. 2.5 cm above mid clavicular line b. 2.5 cm above
sternoclavicular joint c. Not extend to neck
16. Long scenario cytokeratin stain for? Carcinoma
17. Anaesthesia of upper limb by axillary route.. there is intact sensation on
lateral surface of forearm which nerve spared musculocutaneous nerve
18. Hyponatremia most commonly occur in? A. Excessive water intake B.
Hyperosmotic fluid administration c. Post op patient D. Gastro enteritis with
fluid intake (not sure this option was there or not)
19. Effect of vagus on heart rate? A. Contractlity of heart B. Na influx C.k
efflux
3
20. Cephalosporin MOA A. Transpeptidase inhibition B. Dec synthesis of
peptidoglycan
21. Long scenario of rheumatic fever which valve involved commonly mitral
and aortic
22. Scenario of sickle cell disease hbSS
23. Opportunistic organism after spleenectomy streptococcus pneumonia
24. Anterior cruciate ligament damage direction of tibia over femur?
Anterior
25. Narrowest part of paediatric airway cricoid
26. Regarding vertebral column cervical vertebra are 7
27. Nerve involved in bells palsy 7th cranial nerve
28. Dilatation of aortic arch will compress A. Left bronchus b.Trachea C.
Oesophagus D. Left recurrent laryngeal nerve
29. Axillary artery formed by basilic vein and vena commitants of brachial
artery
30. Jejunum supplied by SMA
31. Regarding vessels of lower limb femoral vein is lateral to femoral canal
32. Light thrown in right eye direct reflex is present but absence of indirect
light reflex. Which structure is involved? A. Right occulomotor nerve B. Left
occulomotor nerve C. Pretectum
33. Beta endorphin present in A. Hypothalamus B. Medulla C. Pituitary
34. Patient comes with tender right hypochondrium stool with occult blood
on colonoscopy ascending colon was involved showing ulcers while other
parts of colon spared. Diagnosis? A. Necrotizing vasculitis B crypt abcess C.
Ent amoeba histolytIca D. Carcinoma
35. Child with history of bed wetting though toilet trained. Investigation
done at one side duplication of ureter was found
4
and one of them was opening in vagina cause? Early division of ureteric bud
36. Motor end plate = release acetylcholine
37. Coal mining sand blaster silicosis
38. Anxiety is decreased by activation of which receptors? A. GABA b.
Glutamate c. Dopamine d. Glucocorticoid e. Nicotinic cholinergic
39. A person had injury which did not heal after 1 month what is missing a.
Skin appendages b. Granulation tissue c. Fibroblast d. Underlying connective
tissue
40. Air borne transmission occurs in? A. Diphtheria b. Anthrax c. T B
41. Lymph flow will be decreased by increase of A. Haemorrhage b. Increase
arterial pressure c. Venous pressure d. Increase capillary permeability e.
Tumor
42. Air droplet infection can spread upto distance of? A. 1 2 feet b. 3 6 feet
43. Repeated bcq of dopamine half life 9 minutes
44. What structure Is Present In Floor Of Inferior Horn Of Lateral ventricle
medially A. Insula b. MCA c. Caudate nucleus d. hippocampus
45. A person can't recognize right half of body /world where is lesion? A.
Premotor area b. Cerebral cortex c. Neocortex d. Somatosensory association
Area
46. Characteristics role of prostaglandin E2 D2 a. Vasodilation b.
Vasoconstriction c. Increase permeability
47. Normal platelet Function required for? A. Clot retraction b. Clot
organization c. Clot formation
48. Herring bruer inflation reflex inhibits ? A. Apneustic centre b. Pneumatic
centre c. Ventral respiratory group d. Dorsal respiratory group
49. Brain bridge reflex function increase heart rate
5
50. GFR A. Increase with moderate efferent arteriole constriction B. Increase
with severe efferent arteriole constriction C. Hydrostatic pressure in
bowman capsule is only factor which it depends on
51. Surfactant A. Produced by type 3 alveolar cell B. Dec action in DM C.
Works better with lecithin in myrintase D. Inc amount of glycerol
phosphocholine
52. Heart sound difference between s1 and s2? A. High duration b. High
pitch c. High intensity
53. Repeated bcq of rectal prolapse trichuris trichuri
54. Right lung different from left lung A. 12 broncho pulmonary segment b.
Impression of ventricles C. Larger in size d. Lighter in weight
55. HR 75 beats/min PR is 0.3.. if HR inc 225 then PR is 0.1
56. Dorsal column damage which will least affected A. Touch b.
Proprioception C. Astrogonia D. Vibration E. Increase threshold for sensory
stimuli Ans C
57. A pregnant lady presented with lower abdominal pain.. on ultrasound
placenta attaches to myometrium but does not penetrate through it?? A.
Placenta accreta B. Placenta increta C. Placenta percata D. Placenta previa
58. Pinworm infection A. Ingestion of egg form B. Ingestion of larva form
59. Atrial repolarization on ECG seen in A. 1sT Degree Heart block B. 3rd
degree heart block C. HTN
60. QRS complex occur in A. Ventricular systole B. Ventricular depolarization
61. Slowest growing malignant thyroid tumor papillary
62. Steady pressure is detected by A. Meissners B. Ruffinis C. Pacinian
63. St elevation in Leads 11 111 AVF shows inferior wall MI
6
64. Pt with AML undergo bone marrow transplant presented with
pneumonia histology shows large cell with intracellular inclusion.. cause is ?
A.CMV B. Candida C. Pneumocyst
65. Which of following act through Camp mechanism? A. BNP B. ANP c. ADH
on DCT d. ADH on blood vessel Ans C
66. Spermatogenesis A. Primary oocyte form secondary oocyte B. Secondary
oocytes form spermatids C. Spermatids form spermatozoa
67. Hormone responsible for ductal growth and fat deposition in breast A.
Progesterone B. Estrogen C. Prolactin
68. Erythropoietin secretion is inhibited by A. Cobalt B. Hypoxia C.
Theophylline
69. Which toxin responsible for scarlet fever A. Exotoxin B. Endotoxin C.
Erythrogenic
70. Patient with malaise and raised blood pressure, died b/c of
haemorrhagic stroke in basal ganglia on autopsy bilateral small kidneys with
petechial haemorrhages hyperplastic arteriosclerosis and fibrinoid necrosis
diagnosis? A. Fibromuscular dysplasia B. DM C. Systemic sclerosis
71. Old male patient has history of headache in temporal region there is also
thick cord like vessel in that area. On biopsy giant cells are seen? A. ESR
more than 110 B. Steroid not effective
72. Saliva prevents iron utilization by microbes via? A. Lysozymes b.
Lactoferrin
73. Which corneybacterium is common amongst humans diphtheria
74. Vagotomy done A. Decreased gastric acid and pepsin secretion B. Inc
gastric acid and pepsin secretion
75. Which is more in dialysing fluid as compare to plasma glucose
76. Which inhibits stomach emptying CCK
77. Decreased gastric secretions by SECRETIN
78. HLA DR4 rheumatoid arthritis
7
79. Ankylosing spondylitis HLA B 27
80. Antibody receptors are .. hypervariable regions of H and L chains
81. Alpha adrenergic causes mydriasiS
82. Eye infection diagnosed as HSV conjunctivitis treatment trifluridine
83. 5 year old child was diagnosed with polyp and he was having rectal
bleeding. Polyp was removed from rectum by polypectomy what type of
polyp it could be ? A. Hypertrophic B. hyperplastic C. Inflammatory D.
Haemorrhagic E. Polyp adenoma
84. Patient with diagnosis to have stroke.. on CT there was pale infarct..
what could be cause? A. Hypertensive stroke B. Emboli / ischemic stroke C.
Rupture of berry aneurysm D. Atherosclerosis E. Rupture of arteriovenous
fistula
85. Hb 2nd trimester form from? A. Spleen B. Liver
86. Bundle of his supplied by RCA
87. Right haemoglobin dissociation curve occur by dec in ? A. Temp B. 2 3
bpg C. Ph D. Co2
88. Hanging death due to odontoid process
89. Stimulation of thirst centre by? A. Inc ADH B. Alcohol C. Inc aldosterone
D. Injecting hypertonic solution on osmoreceptors E. Angiotensin 2 Ans E
90. Parasitic infection eosinophilia
91. Man waiting for 60 minutes in moderate hot environment.. how body
will regulate temperature? A. Shivering B erection of erector pili C.
Overactive posterior hypothalamus D. Hypermia E. Vasoconstriction ans D
92. Young married woman delivered a healthy baby boy.. now by which
mechanism size of uterus will be reduced or become
8
normal?? A. Phagocytosis B. Proteolysis C. Necrosis D. Inflammation E.
Apoptosis ans E
93. Which of following is rapidly adapting? A. Ruffinis B. Golgi tendon C.
Krause receptor D. Merkel disc Ans B no option of pacinian
94. Thromboxane A2 produced by? A. Platelets B. Basophils C. Histamine D.
Mast cell E. Endothelial cell Ans A
95. Most common anomaly of great vessel? A. TOF B. VSD C. ASD D. PDA Ans
D
96. In cardiac achalasia what found? Absence of fibbers of vagus nerve
97. About topically applied drugs what best describes it? A. Cornea is
hydrophilic B. Cornea is hydrophobic C. Drug must be water and lipid soluble
D. Absorption surface of cornea is 2X Ans C
98. Pneumatic bones are found in? A. Face B. Upper limb c. Vertebral disc D.
Lower limb Ans A
99. Ca channel blocker act on SA node -ans was verapamil
100. In hypovolemic shock thirst is increased by? A. Angiotensin 1 B.
Angiotensin 2 C. Renin D. Aldosterone E. ADH Ans B
101. 20 year old asthmatic patient finding on spirometry vt 450 fev 2000 fvc
3000
102. Graded potential has these features A. Is decremental B. Is non
decremental C. Saltatory conduction D. Follow all on none Ans A
103. Immunoglobulin produce at (it was not by) A. Plasma cell B. Spleen C.
Bone marrow D. Thymus
104. GFR : A. Inc by hydrostatic pressure in Bowman capsule B. Inc by
afferent arteriole constriction C. Inc by efferent arteriole constriction An CC
9
105. Urine hyperosmotic plasma Na value dec cause is ? Inc ADH (this stem
was Confusing So not sure about ans)
106. Renal disease crescents are formed due to??? I marked mesangial +
endothelial other options couldn't recall
107. On histo slide there is number of striated Ducts mostly serous acini
were seen??? A. Parotid gland B. Submandibular C. Sub lingual ANs A
108. Left geniculate body lesion ?? A. Left homonymous hemianopia B. Right
homonymous hemianopia C. Bitemporal D. Binasal E. Total Ans B
109. Submandibular gland divided by which muscle?? Ans mylohyoid
110. Urinary bladder separated from rectum by? Denoviler fascia
111. Patient has fever for one week on auscultation there is murmur on left
of sternum ? A. Pericarditis B. Pleural effusion C. Parietal layer... this stem
was little bit change and ans was C
112. HDL =high protein
113. Preload indicator = LV EDV
114. Pacinian = vibration
115. Merkel diverticulum =remnant of vitelline duct
116. Primary active transport characteristic finding A. Uphill B. Cleavage of
ATP for energy C. Carrier protein Ans Bb
117. Hamartoma =unorganized tissue
118. Antigen presenting cells and high Phagocytic ? A. Monocyte
macrophage B. Eosinophils C. Neutrophil Ans AA
119. Ranitidine less CNS side effect than cimetidine
120. Long thoracic nerve supply =serratus anterior
121. Inc vascularity in Sarcoma
122. Metastasis =e cadherin Loss
123. During deep inspiration = inc pulmonary compliance
10
124. Haemolytic anemia =un conjugated hyperbilirubinemia
125. Partial thyroidectomy done for ?? To save parathyroid gland
126. First branch of abdominal aorta ? Inferior phrenic
127. FFP given for? Warfarin toxicity
128. Internal carotid lies first lateral to external then post and then medial
129. Which of following is appropriate for foetal skull? A. Ant fontanell
formed by 3 bones B. Vault is more than face i marked A..
130. True about bronchioles?? A.Developed prenataly B. Have no cartilage in
walls Ans B
131. Glucornidation of drug causes ? A. Activation of drug B. Lipid soluble C.
Water soluble and inc renal clearance Ans C
132. EBV causes = burkits lymphoma
133. Selective cox 2 = meloxicam
134. Morphine =in terminal cancer patient
135. GI bleed DOC ? A. Inj octeotride B. Tranxemic acid Ans A
136. Inverse square root of variance is?? Ans standard deviation
137. Another Q of standard deviation is ? But there was not option of
inverse square root. Only square root of variance was option
138. Dr should start conversation with patient = asking name age
139. Superior colliculus ? A. Vision B. Auditory ans A
140. Choroidal artery branch of = internal carotid
141. Which pass through cavernous sinus =abducent nerve
142. Mass moves with swallowing due to = pretracheal fascia
143. Leukocyte adhesion molecule? LFA 1
144. Lymphatic related = cystic hygroma
145. Sympathetic muscranic ? A. Sweat gland B. Adrenal medulla Ans A
11
146. Taste sensation= tractus solitarus
147. Philadelphia chromosome = CMl
148. Scenario of SLE = AntiDs DNA
149. Rete ridges? A. Verrucous B. Squamous Ans B
150. Inverse reflex is ? A. Is polysynaptic B. Is disynaptic involving 1 neuron
C. Monosynaptic ANs B
151. Situational crises
152. Left deviation ? A. RBBB B. Right sided MI Ans B
153. Hyperkalemia causes = inc aldosterone
154. Thiazide causes =hypokalaemia
155. Patient present with dizziness vertigo polydipsia polyuria polyphagia
where is lesion? A. Pituitary gland B. Hypothalamus C. Alpha cell of pancreas
D. Beta cell of pancreas Ans D
156. Cause of edema in CCF?? A. Inc permeability B. Inc hydrostatic pressure
C. Dec oncotic pressure Ans B
157. Fibroblasts = amorphous matrix
158. Fastest fibres = purkinji fibres
159. Cause of atheroma?? A. Diastolic HTN B. Systolic HTN C. Alcohol
160. Sympathetic stimulation causes = inc blood flow nd inc GFR
161. Vit D 3 correct statement? A. Animal origin B. Plant origin C. 1 125vit D
ans C
162. Function of vit D = inc renal Ca reabsorption
163. C wave occur during ? A. Opening of semilunar valve B. 1st heart sound
ans A
164. Adrenergic stimulation causes = gluconeogenesis and glycogenolysis
165. Tumor immunity = NK cell
12
166. Dexamethasone as compare to hydrocortisone? A. Immunosuppression
B. Hyperglycemia Ans A
167. Central chemoreceptors respond to ?? A. CSF ph B. Pco2 of cerebral
vein C. Dec pO2 in arterial blood D. Inc Co2 in arterial blood Ans D
168. Case of rheumatic heart disease most common cause = streptococci
viridian
169. Stratified columnar epithelium found in ? A. Esophagus B. Stomach Ans
esophagus other options were wrong
170. Histamine releasing opioid? Ans morphine
171. Endotoxin ? A. Release after bacteria death B. Strongly antigenic Ans A
172. Injury to lateral geniculate body causes? A. Increases appetite B dec
hunger C. No effect on hunger Ans B
173. Radiation cause malignancy= 5 10 years

174 Eclamptic patient on OT table drug of choice for HTN? A. Sodium


nitropruside B. Iv hydralazine C. Methyldopa Ans B
175 4 year old girl complains that she is abnormally tall and also secretion
from her mammary glands on x ray growth of pituitary sella turcica which
cell increase? A. Chromophobe b. EosinophiL c basophils

this key is not 100 % correct

Gynae obs 2 march evening 2017


1. Medial relation of floor oflateral ventricle
a. Hippocampus (answer)
b. Caudate nucleus
2. Swelling on front of neck move with swallowing.

a.pretacheal fascia answer


3. endotoxin released due to
a. when bacteria are lysed ans
b. antigen
4.whip like movement
a. uterine tube
5. stress related increase glycogenolysis
a. cortisol ans
b. glucagon
c. epinephrine
6. semen analysis abnormal thing
a. sperm count less then 20 million
7, newborn with meningitis
a. Streptococcus
b. Agalactea

8. longest incubation period


a. hep b I did
b. hiv
9. 90% of anthrax have
a. skin disease
10. study done on male patient where STD with penile ulcer compared to non
penile ulcer on
Investigation gonococci was negative. Whats the cause of STD
a. Chlamydia/

11. risk of malignancy


a. more then 10 years
12. main uterine support.
a. tranverse cardinal ligament.
13. case of burns in which skin graft not needed decided on
a. skin appendages
b. granulation tissue
c. connective tissue I did not sure
14. regarding bronchiole
a. absent cartilage ans
b. bronchiole completely develop prenatally
15. long scenario of laparptomy in which surgeon see chocolate cyst treatment of
choice.
a. danazole
16. during parturition enhance myometrial activity is due to increaselevel of
a. prostaglandin
b. oxytocin ans
17. about breast
a. divided into 15-20 lactiferous duct
18. maximal increase in extracellular volume or full body volume or full body
a. hypertonic saline
19. a boy who drink 2 litre distilled water will have (bcq fro brs physiology)
a. inc ecf
b. dec osmolarity both look right but I did this one
20. old scenario of earth quake
a. situational crises
21. thirst will be stimulated by
a. angiotensin 2 ans
b. aldosterone
22. standard deviation (rabia ali bcq)
a. is sq of variance
b. is sq root of variance ans
c. mean is ot considered
d. median is the central part
23. sq root of variance is (rabia ali bcq)
A. variation
b. standard deviation ans
c. median
d. accuracy
24. the best approach to patient is (rabia ali )
a. start with open ended question sart with close ended question start by asking
presenting
complains
b. Start by asking his name age (ans)

25. during sexual excitement in female fluids are secretedby vaginal walls mainly
due to
a. adrenaline
b. bombesin
c. GnRh
d. relaxin
e. parasympathetic stimulation answer ( asim n shoaib gynea bcq but e option
that is VIP was
replaced by parasympathetic stimulation ) cpsp trick
26. second messenger cAMP
a. ADH 2
27. part of adrenal gland responsible for regulating eosinophils and lymphocytes
a. zona fasciculate ans
b. zona glomerulosa
c. adrenal medulla
28. cystic hygroma related to
a. disease of lymphatic vessels
29. patient of IHD and essential HTN having pedeal edema reason
a. inc hydrostatic pressure ans
b. dec oncotic pressure
30. platelet function
a. clot formation
b. clot retraction
c. clot organization
31. female with fever since 10 days got chest pain on auscultation some rustling
sound audible (point
Of confusion by cpsp)
a. Myocardium ans
b. Pericardium

32. highest proyein content


a. HDL ans
b. LDL
c. chylomicrons
d VLDL
33. best function to check renal function .
a. serum urea
b. creatinine clearance I did
c. serum creatinine
34. mid trimester erythropoisis
a. liver ans
b. yolk sac
c. spleen
35. most common big vessel anamoly
a. PDA answer
b. situs inversus
c. ASD
36. vitelline duct remnant
a. meckel diverticulum
37. lymphatic flow decrease in
a. inc in venous pressure I did
b. inc arterial pressure
c inc protein leakage
39. which will increae gfr
a. moderate constriction of efferent arterioles
b. severe constriction of efferent arteriole
40. similarity b/w nitroglycerine histamine and isoproterenol (repeated from 1
march paper)
a. tachycardia
b. postural hypotension I did
41. marked inc in CO
a. pregnancy I did
b. exercise
c, anxiety
c. Extremes of temp

42. esmolol
a. dec elimination in renal failure ans
b, dec elimination in liver failure
43. during laparotomy near ovarian fossa which structure is prone to be injured
a. internal iliac artery
44. which cell is present on leucocyte
a. LFA 1
b. VCAM
45. reversible change in which there is change in size and polarity
a. dysplasia
46. 4 yrs old child having per rectal bleedingwith mucus biopsy showssome
inflammation
a. hemartomatous
b. hyperplastic
47. most common cause of ca cervix
a. hpv ans
b. hiv
c. hsv
48. cell which present antigen or tumour (dont remember exactly ) tp cytotoxic
CD8 cells
a. macrophage
49. primitive cell for killingtummor cells
a. NK cells ans
b. CD8
50. thromboxane present in
A, platelet
51. HIV positive but AIDS not proven CD4 cell 800 got pyogenic SOM the causative
organism
a. pneumonitis juruveci
b. streptococcus pneumonia
c. staph aureus ans
52. EBV is diagnosed in which of the following disease
a. burkits lymphoma
53. amyloid is deposited mainly
a. kidney
54. most commonly genetic anamoly
a. multifactorial
55. lowest limit of spinal cord in children
a. lowe border of L 3
56. person had swelling on upper arm due to some injury swelling remained 3
months but resolve
In 1 year what was cause of cause of swelling
a. Hypertrophy ans
b. Hyperplasia
c. Neoplasia
d. Dysplasia

57. blood supply of bundle of HIS


a. RCA
58. S1 ans S2 are different what is specific in former
a. increase duration ans
b. increase in frequency
c. increase in pitch
59. alpha methyl dopa MOA
a. inhibits angiotensinogen to angiotensin to angiotensin
b. it inhibits norepinephrine for post ganglionic neuron to act
60. maximum anti cholinesterase inhibitor which can travel brain
a. physostigmine ans
b.. neostigmine
c. pyridostigmine
61. case of pulmonary tb with cavitation shows calcification. Reason ??
a. dystrophic calcification ans
b. metastatic calcification
62. chances of neoplasia
a. protooncogenes ans
b. apoptosis
63. ectopic thyroid harmone produced by
a. sq cell ca of larynx ans
b. small cell ca
64. infant diagnosed to have worm infestation physician did CBC which
conformed diagnosis by
Presence of
a. Eosinophilia
b. IDA

65. Intrauterine infection in amniocentesis


A, IL -6
66. scenario of pregnant lady which develop some skin change just like her uncle
who is HCV +
And cirrhosis which are those changes?
A. Spider telengectasia and palmer erythema
B. Spider telengectiasisand hyperpigmentation I did
67. subacute endocarditis
a. streotococus viridian
68. Ejection fraction calculation .. CO 5.5 litre
HR 70
EDV 160
a. O.5 ans
b. 0.55

69. c wave shows


a. closure of semilunar valve
b. opening of semilunar valve
c. closure of tricuspid mitral valve ans
d. opening of tricuspid and mitral valve
70 . fetal hb
a. Alpha 2 beta 2
b. Higher affinity or content of o2 as compared to adult ans

71. scenario in which ther was RBC slide in which they show thin streak of iron
a. hypochromic and microcytic
72. band syndrome.. cause ?
a. oligohydromnios
73. increase homocystinuria
a. deficiency of vit b 12
74. most imp investigation for IDA
a. serum ferritin
b. serum iron and TIBC
75. ca breast diagnosed surgical removal done a small mass removed treatment
after
After surgery
a. Tamoxifen

76. cervical mass 3.5 cm lymph node unilateral palpable and metastasis
a. T2 N1 M1
b. T1 N2 M1 ans
77. 25 yrs old mother of two children now having sec amenorrhea she noticed
milk discharge from
Ophthalmic changes also present. Diagnosis?
a. Hyperprolactinoma

78. two tailed test???


Forgot options
79. swelling on neck which moves on respiration . infection is in ??
a. pretracheal infection
80. stratified cubital epithelium
a. large glands of salivary glands
81. increase in age will decrease
a.HR I DID This one
B. MAP
C. RV
D. TVC
82. deep inspiration causes?
a. inc compliance in pulmonary vasculature
83. gamma globulin form in
a. plasma cells
84. there was short case of acid base imbalance ph 6.3 CO2 50 HCO3 30 .
diagnosis?
A partially compensated respiratory acidosis
85. tractus solitarious
a. taste pathway
86. fast adaptation
a. golgi tendon
b. merkels
c. ruffinis
d. hair containing parts of body
e. Krause cprposcles
87. pars distalis contain
a. pituicytes
88. product of GFR and plasma conc of substance
a. FF
b. filtration co efficient
c. filtration load ans
89. hering buere reflex will lead to inhibitor of
a. dorsal neurons of central resoiration ans
b. apneustic centre
c. pneumatic centre ans
90. central receptor respond to
a. inc H+ conc in arterial blood
b. inc CO2 in arterial blood
c. inc H+ in cerebral interstitial fluid ans
91. vit D 3 is called
a. plant origin
b. animal origin ans
c, 1,25 (OH)2
92. posterior surface of bladder
a. denonvillers fascia
93. in partial thyridectomy posterior thyroidectomy done carefully because??
a. parathyroid gland ans
b. recurrent laryngeal nerve
c. superior thyroid artery
94. medial relation of inferior horn of lateral ventricle
a. caudate nucleus
b. hippocampus ans
95. over extension of neck lead to
a. UMN lesion in upper limb
b. LMN lesion in upper limb ans
c. UMN in loWer limb
d. LMN in lower limb
96. response 0f tapping of patellar tendon by tunning fork will lead to
a. contraction of quadriceps femoris muscle
97. very very big scenario of qualitative surveydone which consist of collection of
data of false
negative pap smear report .then researcher took half sample of false negative
report of
prior year and compare it with I dont remember any more
a. Not covering lid that makes sample dry ans
b. There wer other strange option I cant recall

98. bias study


a. sample error ans
99. deep inspiration is associated with increases in spinal anesthesia . reason ?
a. due to cardiac depression ????
100. rete ridges waala repeated question
a. sq cell carcinoma.
101. 18 yrs old girl height 150 cm wt 70 kg hair distribution on biody FSH 52 and
LH 40 inc
testostero inc estrogen diagnosis ?
a. PCOD ans

102. 25 years old lady with sec amenorrhea with raised FSH and LH . what is
cause?
a. autoimmune disease
103. OCP discontinued due to presence of
a. migraine
b. cervical neoplasia ans
104. death due to hanging due to
a. odontoid process of C2
105. a boy standing in hot sunny day heat loss is by
a. activation of post hypothalamus
b piloerecti
c.Cutaneous hyperemia ans
106. apex of lung reach
a. 2.5 cm above sternoclavicular joint ans
b. 4cm above sternoclavicular joint
c. 2.5 cm above midclavicular joint
107 blood supply od bundle of HIS
a. RCA

108. insensible water loss


a. 600-800 ml
109. how good a test at correcting excluding people without the condition is
feature
a. specificity ans
b. sensitivity
c. negative predictive value
d. positive predictive value
110. about primary active transport
a. uphill movement ans
b. downhill movement
111. most common organ involved in amyloidosis
a. kidney
112. parent bring their baby with difficulty in taking milk and nasal regurgitation
a. palantine shelves
113. thyroid harmones increase in oregnancy due to
a. TBG Ans
B. TSH
c. inc harmone synthesis
d. fetal thyroxine level
114. the vitelline ducts form
a. ductus venosus
b. duodenum
c. meckel diverticulum ans
d. portal vein
e. urachus
115. iodination of tyrosine takes place in
a. principle cell ans
b. parafollicular cells
116. inc collagen synthesis which vitamin is required
a. vit c
117. graded action potential
a. all or none
b. incremental ans
c. non decremental
118. decrease lymphatic drainage
a. inc venous obstruction ans
b. heamorrhage
119. longest incubation period
A, hiv ans
b. Hep b

120. in prolong exercise will get glucose from


a. fatty acid from adipose tissue
b. glycogenolysis in liver
121. scenario of pregnancy in which hb in 1st trimester was 11.5 but at term it
become
9.5 what is cause?
a. physiological change
122. central tendency
a. 16, 18 20, 22, 24 median in this data is 20
123. autosomal dominant
a. only females are effective
b. complete penetrance ans
c. heterozygous
124. hereditoryspherocytosis
a. defect in memb cytoskeleton
125. fibroblast
a. amorphous substance
126. best investigation to check renal function
a. serum creatinine
b. serum urea
c. creatinine clearance ans
127. mid trimester erythropois is
a.
128. post adrenergic / sympathetic
a. miosis
129. lady with severe abd pain diagnosed as mass in lt fallopian tube WBC inc on
biopsy
a. monocyte/macrophage ans
b. fibroblast
130. very thin wall tube with endothelium and smooth muscle 3-4 layers which
one is from the
Following.
a. Venules
b. Arteriole
c. Medium size arteriole

131. increase causes of artherosclerosis


a. type b disorder
b. cholesteama ans
132. conscious desire of thirst
a. gastric distention
b. hypovolemia ans
133, aorta
a. Origin of renal artery from L 2
134. sinusoid of liver
Options not remembered
135. lateral boundry of deep inguinal ring
a. conjoint tendon
b. transversalis fascia ans
136. regular insulin is
a. short acting enzymes
b. short acting chain mechanism
137. anterior frontanelle
a. formed by fusion of 3 bones
b. closed by 3rd month
c. vault of skull is more then size of face of body
138. patient with fever with chills and rigors passing black urine . some team
came and took
Test and conform
a. Malaria

139. prostaglandin E2 F2
A. vasodilation
140. diaphragmatic hernia
a. incomplete pleuroperitoneal memb
141. biopsy of gland with serrous acini
a. parotid gland ans
b. mandibular gland
c. sublingual gland
142. how you would identify ketone bodies
a. acetoacetate ans
b. acetyl acetate
143. role of vit D
a. reabsorption in kidney ans
b. iron absorption
c. germinal epithelium
144. a lady went to jeddah got oral mucosal vesicae
A, HPV 1
145. maximum inc in volume
a. hypertonic saline
146. muscle of pelvic diaphragm
a. puborectalis muscle
147. pelvic diaphragm
a. superficial muscle ans other option were not correct at all
b.through which urethra vagina and rectu m passess
148. tall man with normal seminal fluid secretion but azoospermia defect is in
a. sertoli cells
149. sertoli cells produce
a. MIF
150. parents wer worried for thir son about he has difficulty in passing urine
a. ventral shaft of penis
151. cervical ca lymph nodes involve
a. int and external iliac lymph nodes
152. nerve involved in cavwernous sinus
a. abducent nerve
153. disease in progress to terminate in Pakistan
a. polio ans
b. tb
154. reaabsorption in kidney takes place in which segment
a. major calyses
b. straight convoluted tubules
155. epidural block by reaching ischial spine , which nerve is blocked
a. pudendal nerve
156 maximum constriction of ureter is at
a. Vesicoureteric constriction

157. during surgery urinary incontinence due to paralysis of some muscle in which
region
a. urogenital diaphragm
158. placenta attach but not invade myometrium
A. placenta accretta ans
b. placenta incretta
c. placenta pancretta
159.

Gynaecology and obstetrics morning paper Thursday March 2,


2017 by Dr. HinaHammad.
( Kindly remember me in your precious prayers).

1. Basilar artery gives off

A. Vertebral arteries

B. Posterior cerebral arteries

C. Anterior cerebral arteries

Ans. B

2. When the vagal supply is cut. Which group of muscles most affected

A. GIT muscles
Ans.A (Cant bring back other options in mind, vague recall. )

3. Regarding megaloblastic anemia

A. Oval macrocytosis

Ans. A (All other options were completely wrong)

4. Regarding pernicious anemia

A. Anti-parietal cell antibodies

B. Megaloblastic anemia

C. Iron deficiency anemia

Ans. A

5. hCG produced in late pregnancy by

A. Sytotrophoblast and fetal endotheliam

Ans. A

6. Cells with anti-tumor activity

A. Natural killer cells

Ans. A

7. Ciliary movements

A. Actin/ Myosin

B. Intermediatefilaments/Desmin
C. Microtubules/Denein arm

Ans. C

8. Confidentiality can be breached when

A. Patient authorisez you

(All other options were wrong so couldn't remember)

8. Coefficient of variance indicating prefect positive correlation

A. +0. 5

B. +1

C. -1

D. -0. 5

E. 0

Ans. B

Explanation: Perfect negative correlation r=-1, perfect positive correlation r=+1,


no correlation r=0

9. Medical ethics

A. Required for Pakistan medical and dental council

B. Code of conduct of doctor

C. Study of moral aspects of doctor's life

Ans. C

10. Aponeurosis of transversus muscle

A. Forms inguinal ligament


B. Conjoint tendon

C. Lacunar ligament

D. Superficial inguinal ring

Ans. B

11. Regarding pituitary gland

A. Lateral to cavernous sinus

B. Derived from endoderm

C. Lies in diaphragm sellea

D. Pituitary tumor causes junctionalscatoma

Ans. D

12. Regarding bronchopulmonary segments

A. Conical in shape

B. Has an independent vein

C. Has an independent somatic nerve

D. Supplied by tertiary bronchioles

E. Supplied by terminal bronchioles

Ans. D

13. Diphosphoglycerate

A. Product of glycolysis

B. Increased in stored blood


C. Increases the affinity of hemoglobin for oxygen

D. Concentration increases with increase in temperature

E. Shifts oxygen-hemoglobin dissociation curve to the left

Ans. D

( There are two types of diphosphoglycerate:

-1,3diphosphoglycerate and 2,3 diphosphoglycerate.

-1,3diphosphoglycerate is an intermediate in glycolysis.

-2,3diphosphoglycerate is decreased is stored blood and decreases the affinity of


hemoglobin to oxygen. It shifts oxygen-hemoglobin dissociation curve to the
right.)

14.

A patient has pain on nose and the area between mouth and orbit and temporal
region. Which of the following nerve involved

A. Cervical branch of facial nerve

B. Maxillary branch of trigeminal nerve

C. Mandibular branch of trigeminal nerve

Ans. B

15. Which of the following metabolite of vitamin D can be used to assess the level
of vitamin D

A. 25 hydroxyvitamin D

B. Cholecalciferol
C. 1, 25 dihydroxyvitamin D

Ans. A

16. Radiation induced carcinoma after

A. 8 years

B. 2 years

C. 10 years

D. 1 year

E. 6 years

Ans. C

17. Neck of blader and urthra nerve supply

A.Sympathethtic L1- L2

B. Pelvic splancnic nerves S2-S4

Ans. B

18. During inspiration

A. Venous return decreases

B. Alveolar pressure increases

C. Heart rate increases

D. Venous return increases

Ans. D
19. Regarding testosterone

A. Produced by leydig cells

B. Intradermal route less effective

C. Something about oral route

Ans . A

20. Which of the following will prevent involution of corpus luteum.

A. Endogenous chorionic gonadotropin

B. Exogenous chorionic gonadotropin

C. Progesterone

Ans. C

21. Safety margin

A. Therapeutic index

B. High with digoxin

Ans. A

22. A lady has 28th day cycle. She has increased estrogen level but no Increase in
progesterone for the last three days.

A. Cessation of menstruation

B. Start of menstruation
C. Secretary phase

D. Ovulation

E.Regression of corpus Luteum

Ans. b

23. Regarding levatorani

A. Contains ischiocavernosis, corpus spongiosum and bulbospongiosis

B. Tripartite muscle pierced by urethra, vagina and anal canal. (Complete other
options if you people remember).

Ans B

24. Injury to spinal cord above sacral segments

A. Spastic bladder

B. Atonic bladder

C. Neurogenic bladder

D. Automatic bladder

Ans. D

25. NON-CONTRACEPTIVE benefit of contraceptive pills

A. Prevention of ovarian cancer

B. Prevention of breast cancer

C. Prevention of thromboembolism

Ans. A

26. Inherited form of endometrial carcinoma is associated with


A. BRCA

B. Breast cancer

C. HNPPC

Ans. C

Mean systemic filling pressure determines

A. Venous return

B. Right atrial pressure

C. Cordiac output

D. Decreases pulmonary resistanse

Ans. A

27.Regarding micturition reflex

A. Sacral segments of spinal cord S1 and S2

B. Sacral sagments of spinal cord S2 and S3

C. Lumbosacral segments of spinal

D. Controlled by midbrain

E. controlled by cerebrum

Ans. ?????

28. Systemic antifungal therapy is CONTRAINDICATED in

A. Positive blood culture by IV catheter

B. Candida isolated from two sources in body


C. Oral thrush and oral candidiasis

D. When only Candida is isolated from fluid

E. Aspergilus infection

Ans .A

29. Strong VAGAL stimulation

A. Decrease heart rate

B. Increase heart rate

C. Decrease myocardial contractility

Ans. A

30. Common in facilitated and co-transport.

A. Both for inorganic ions

B. Uphill

C. Require carriers for transport

Ans. C

31. Minimum amount of urine produced in dehydration to remove waste


products from body.

A. 500-600 ml

B. 200-400 ml

C. 1000 ml
D. 800 ml

E. 2000ml

Ans. B

32. Hemolytic anemia occurs after a miss-match blood transfusion. It can be


diagnosed clinically

A. Bilirubinuria

B. Hemoglobinuria

C. Severe cyanosis

Ans. B

33. Moderate hemorrhage can be diagnosed clinically as

A. Postural dizziness

B. Supine hypotension

C. Postural hypotension in 35 years old

D. Postural hypotension in 65 years old

E. Supine tachycardia

Ans. A

34. Corticospinal tract

A. Contains fine touch

B. Decussate in midbrain

C. Carry only motor fibers


D. Unmyelinated

Ans. B

35.Hyperosmoticoverhydration

A. Excess ADH

B. Bronchegenic tumor

C. Excessive use of diuretics

D. Cushing syndrome

Ans. ???

36. HIV associated nephropathy

A. FSGN

B. Membranous nephropathy

C. Membranoproliferative GN

D. Interstitial nephritis

Ans. C

37. Protooncogenes converted to oncogene

A. Point mutation

B. Amplification

Ans. A

38. A man has sweating on a hot day. He has?

A. Positive free water clearance B. Negative free water clearance


C. ECF hypoosmolarity

D. Urine hypooamolarity

Ans. B

38. Duct of sylivus between

A. Third and fourth ventricle

Ans. A

39. TNF production by

A. Helper T cells

B. Monocytes/macrophages

C. Cytototoxic T cells

Ans. B

40. Tickle sensations

A. A alpha fibers

B. A delta fibers

C. A beta fibers

Ans. ??

41. About right principal bronchus

A. More wider than left

B. Runs obliquely

C. More liable to obstruction

D. Longer than left


Ans. A

41. Pacinian

A. Unmyelinated fibers

B. Unencapsulated

C. Detect low frequency vibrations

D. Detect high frequency vibrations

Ans. D

42. Most likely about tuberculosis

A. Foul smelling sputum

B. Wheezing never occurs

C. Never involve pleura

D. Involve posterior segment of upper lobe

Ans. D

43. Metabolic acidosis with normal anion gap

A. Starvation

B. Diabetic ketoacidosis

C. Chronic renal failure

D. Carbonic anhydrase inhibitors

Ans. D
44. While doing research, statistical tests are done for

A. The results are true

B. The results are clinically significant

C. To accept the placebo

D. Minimize random or experimental error

Ans. ???

45. Most likely about iron absorption

A. Ferric form readily absorbed than ferrous

B. Most absorption occurs in distal ileum

C. Heam is readily absorbed

D. Dietary iron is readily absorbed

Ans. D

46. Left shift of oxygen-hemoglobin dissociation curve

A. Anemia

B. Hypothermia

C. Increased temperature

D. Acidosis

E. Ageing

Ans. B

47. When each person has equal chance of being selected in a sample. This
sampling is called?
A. Stratified sampling

B. Random selection

Ans. A

48. Cimetidine

A. Hepetic enzyme inhibitor

Ans. A

49. Inherited by mother only

A. X genes

B. Mitochondrial RNA

C. Ribosomes

D. Endoplasmic reticulum

E. Golgi complex

Ans. B

50. Thalassemia

A. Abnormal globin chain production

B. Unequal chain ratio

Ans. A

51.For the nutritional assessment of a patient, we assess nutrition from his weight
loss and test for

A. Minerals

B. Vitamins
C. Trace elements

D. Albumin and TIBC

E. Lipids

Ans. D

52. Which globulin absent in intrauterine life?

A. Alpha ()

B. Beta ()

C. Gamma ()

D. Delta ()

E. Epsilon ()

Ans. B

53. The primary oocytes complete their first meiotic division

A. Before ovulation

B. 3rd month intrauterine life

C. 6th month intrauterine life

D. After fertilization

E. Before fertilization

Ans. A

54. Pap-smear prevention level

A. Primary
B. Secondary

C. Tertiary

Ans. B

55. The undifferentiated germ cells in semineferous tubules

A. Sparmatids

B. Type Aspermatogonia

C. Type B spermatogonia

(A spermatogonium (plural: spermatogonia) is an undifferentiated male germ cell.


Spermatogonia undergo spermatogenesis to form mature spermatozoa in the
seminiferous tubules of the testis.

There are three subtypes of spermatogonia in humans:

Type A (dark) cells, with dark nuclei. These cells are reserve spermatogonial stem
cells which do not usually undergo active mitosis.

Type A (pale) cells, with pale nuclei. These are the spermatogonial stem cells that
undergo active mitosis. These cells divide to produce Type B cells.

Type B cells, which divide to give rise to primary spermatocytes.

Ans. B

56. A baby with hypoplastic nails, microcephaly, clept lip, mother during her
pregnancy was exposed to?
A. Fetal-alcohol syndrome

B. Phenytoin

C. Warfarin

D. Hypervitaminosis A

Ans. B

57. Regarding blood supply of bones

A. End of Long bones supplied by epiphysial artery

B. Flat bone supplied by Nutritional artery

Ans. A

58.function ofVitamin A?

A. Forms germinal epithelium

B. Maintains Integrity of skin epithelium

Ans. B

59. The most potent anti-oxidant

A. Glutathione

B. Vitamin E

C. Vitamin A

Ans. A

60. Murmur of mitral stenosis is best heard at


A. Left second intercostal space

B. At mitral area

C. Right 2nd intercostal space

D. Left 5th intercostal space

Ans. B

61. Shortest incubation period

A. Measles

B. Rubella

C. Chicken pox

D. Mumps

E. H. Influenza

Ans. ???

62. In liver diseases which enzyme is decreased

A. 5 nucleotides

B. AST

C. ALT

D. ALP

E. LDH

Ans. A
63. Agirl during her menstruation developed high grade fever because she was
using tampons. Microorganism responsible for her symptoms is?

A. E.coli

B. Staphylococcus aureus

Ans. B

64. The most acute effect of radiation

A. Endarteritis obliterans

B. Skin malignancy

C. Lymphoproliferative cancers

D. Desquamation

Ans. D

65. waterintoxiacation clinical sign?

A. Slow pulse

B. Raised JVP

Ans. A

66. A woman feels lethargic and has dyspnea on exertion, tingling sensation in
feet and difficulty walking. Her Hb is 5 6, WBC 3500, platelets 120,000.Diagnosis?

A. Iron deficiency anemia

B. Aplastic anemia

C. Vitamin B12 deficiency

Ans. B

67. A pregnant female needs a lead shield to undergo


A. x-ray chest

Ans. A

68. A female has undergone hysterectomy, now she has pain in flanks and lower
abdomen, oliguria and difficult micturition. Structure damaged?

A. Ureter

Ans. A

69. Fat embolism

A. Fracture of long bones

B. Symptoms appear after 72 to 96 hours

Ans. A

70. After surgery , a patients wound is not healing. She is not taking fruits and
vegetables. What is the cause?

A. Collagen tensile strength is decreased

B. Type 3 collagen is deficient

Ans. B b

71. Hepatitis- B positive when

A. HBsAg and HBcAg positive

Ans . A
72. Hepatitis B is best monitored by

A. ALP

B. AST

C. SGPT

D. LDH

Ans. C

73. Maternal mortality in Pakistan compared to developed countries ?

A. 10 times more

B.100 times more

C. 200 times more

D.1000 times more

Ans. B

74. Epidural space

A. From foramen magnum to L5

B. From foramen magnum to s2

C. From foramen magnum to L1

Ans.

75. Iron deficiency anemia diagnosis

A. Ferritin+TIBC

B. CBC
C. TIBC

D. Ferritin

E. Bone marrow biopsy

Ans. A

76. During exercise the blood flow in all zones of lung is like that of

A. Zone 3

B. Zone 2

C. Combination of zone 2 and zone 3

D. Zone 1

E. Combination of zone 1 and zone 3

Ans. C

77. Regarding pulmonary parameters, what happen to them in pregnancy?

A. Vital capacity decreases

b. FEV1 /FVC ratio decreases

C. Low P02

D. Low PC02

Ans. D

78. During ovulation which is not present ?

A. temperature increase

B. Vacuolization

Ans. B
79. Posterior 1/3 of interventricular septum is involved in MI. Which artery
involved

A. Right coronary

B. Left coronary

C. Marginal

D. LCX

E. Left descending artery

Ans. A

80. Glucose transported through placenta

A. Facilitated diffusion

B. Passive diffusion

C. Simple diffusion

Ans. A

81. IncreasedSpecific gravity of urine, derangements in which function of Kidney


has occurred ?

A. Concentration

B. Filtration

C. Reabsorption.

D. Secretion

Ans. A

82. Mechanism of action of local anaesthesia


A. Blockade of sodium channels

B. Inhibit resting membrane potential

C. Move K outside cell

Ans. A

83. In cerebellar lesion, which one among the following is present ?

A. Dysmetria

B. Resting tremors

C. Muscle paralysis

D. Hypotonia

Ans. A

84.Saw toothed appearance of glands, vacuolization and edematous storma?

A. Secretory phase

B. Proliferative phase

C. Menstrual phase

D. Ovulation

Ans. A

85.Secondaryamenorrhoea history, best investigation?

A. Mid-luteal phase progesterone

B.Estrogen

C. FSH, LH, Prolactin

Ans. C
86. First stage during onset of puberty?

A. Menstruation

B. Breast bud appears

C. Pubic hair

Ans. B

87. The most important function of progesterone is

A. Secretory changes of endometriam

B. Development of alveoli of breasts

C. Menstruation

Ans. A

(Progesterone converts the endometrium to its secretory stage to prepare the


uterus for implantation.)

88. breast innermost is involved in carcinoma. LEAST likely drainage into

A. Anterior axillary nodes

B. Internal thoracic nodes

C. Inferior phrenic nodes

D. Posterior intercostal nodes

E. Supraclavicular nodes

Ans. C???
89.Patient with adrenogenital syndrome

A. High 17-Ketosteroid, high cortisol and low pregnenolone

B. High 17- Ketosteroid, low cortisol and high pregnenolone

C. Low 17-Ketosteroid, high cortisol and high pregnenolone

D. Low 17-ketosteroid, high cortisol and low pregnenolone

E. High 17-Ketosteroid, high cortisol and low pregnenolone

Ans. B

90. One Bar body present in nuclei/cell of

A. Turner syndrome

B. Down syndrome

C. XXY

D. Normal female

E. normal male

Ans. D

91. Strong microbicidal kill

A. Hydrogen per oxide

B. SuperoxideB

Ans. B

92.The most striking histopathologic feature of nasal polyp

A. Squamous metaplasia

B. Ciliated columnar epithelium

C. Lymphocytes
D. Eosinophils

Ans.A or B???

93. undescended testis removed because risk of?

A. Carcinoma transformation

B. Infertility

Ans. A

94. Urothelium in following condition will have great chance to convert to


carcinoma?

A Horseshoe shaped kidney

B. Extrophy of bladder

C. Duplicate ureter

D. Medullary sponge kidney

Ans. B

95. Serous cells

A. Contain mucin

B. Basophilic base

C. Nucleus flat

D. Eosinophilic

E. Nucleus at periphery

Ans. ???

96. Mass felt on pubis and pubic bones malformed. Child getting wet easily
A. Extrophy of bladder

B. Absent mesenchyme

Ans. B

97.A long scenario for Gram-negative rods on urine culture .

A. E-coli

B.Staph-aureas

Ans. A

98. Sexually transmitted disease most commonly by

A. Chlamydia

B. Neisseria gonorrhoeae

Ans. A

99. Cervix lymph drainage

A. Internal iliac nodes

(All the other options were completely wrong)

Ans. A

100. A boy having high grade fever with jugulodigastriclymphnodes enlarged?

A. Parotid gland

B.Pharyngeal tonsils

C. Sub-mandibular

D. Sublingual

Ans. B
101. Variance definition

A. Square of standard deviation

B. Square root of standard deviation

Ans. B

102. Genital triangle Nerve supply lost nerve involved?

A. Genitofemoral nerve

B.Ilioinguinal nerve

C. Hypogastric nerve

D. Pudendal nerve

Ans. D

103. In normal distribution

A. Mean, mode and median coincide

Ans. A

104. New born baby with recurrent infections?

A. IgA difeciency

Ans. A

105. In ARDS, edema is due to?

A. Increased capillary permeability

B. Increased hydrostatic pressure

Ans. A
106.Parturition is initiated by

A. Maternal cortisol
B. Fetal cortisol

Ans. B

107. Angiogenesis in wound healing by?

A. Anti-trypsin

B .Caspase 9

C. Metalloproteinases

Ans. C

108. Parathyroid gland supplied by

A. Thyroid ima and inferior thyroid artery

B. Thyroid ima and superior thyroid artery

C. Superior and inferior thyroid artery

D. Superior thyroid artery

E. Inferior thyroid artery

Ans. C

109. Cytokines cause fever by.

A. Direct action on anterior hypothalamus

B. C-reactive protein

C. By prostaglandin
D. Thermogenesis

E. Neutophil recruitment

Ans. C

110. Parsdistalis

A. Derived from neuroectoderm

B. supplied by superior hypophysial artery

C.Hypothalamic-portal tarct

Ans. B

111. A cells with large granules and larvicidal polypeptides

A.Eosinophils

B. Neutrophils

C. Monocytes

D. Basophils

Ans. A

112. Neutrophils are studied in blood and are found in periphery with oxygen
consumption. This process is necessary for.

A. Phagocytosis

B. Microbicidal killing

C. Adhesion

Ans. B???
113. Which of the following factors are mostly associated with a pathological
process rather than a physiologic process?

A. Epidermal growth factor

B. Transforming growth factor.

C. Fibroblast growth factor

D. Platelet growth factor Endothelial growth factor

E. insulin like growth factors

Ans. B??

114.Exomphalos

A. Defect in anterior abdominal wall

B .Omphalocele

C .Gastroschisis

D. Defect of umbilicus

Ans. A

115. Left renal vein lies posterior to

A. Aorta

B. Renal artery

C. Third part of duodenum

D. Head of pancreas

Ans. A

116. A pregnant female not taking fruits and vegetables


A. Anencephaly

B. Omphalocele

Ans. A

117. cerebellar lesion vill not lead to

A. Muscle paralysis

B. Speech loss

Ans. B

118. Myasthenia gravis hypersensitivity

A. Type 1

B. Type 3

C. Type 4

D. Type 2

Ans. D

119. Joints between vertebral bodies

A. Symphysis

B. Synchondrosis

C. Syndesmosis

Ans. B

120. Vitamin B12 carried in blood by

A .intrinsic factor
B. Transcobalamin 1

C. Transcobalamin 2

Ans. C

121. Most important mediator of systematic symptoms

A. IL 1

B. TNF

C. Interferon

D. IL 6

Ans. A

122. The pain of diseased ovary is felt on medial thigh because of involvement of

A. Femoral nerve.

B. Obturator nerve

C. Pudendal nerve

D. Perineal nerve

Ans. B

123. A person got a stab wound 3 cm vertical on lateral right side of linea alba.
Initially he was fine but then develops hypotension and shock. Which vessel
damaged?

A. Superior mesenteric artery

B. IVC

C. Inferior mesenteric artery

D. Abdominal aorta
E. Ileal branch of superior mesenteric artery

Ans. B

124. Which of the following is from posterior division of sacral plexus

A. Obturatorinternus and nerve to gamellussuperior

B. Common peroneal nerve

Ans. B

125. A lady taking glibanclamide for diabetes develops hypotension and her blood
glucose level was 40. She had acute asthma for which she took some drug. Which
drug interacted with glibenclamide.

A. Sulbutamol

B. Beta blocker

C. Steroids

Ans. C

126. Axon reflex

A. Triple response

Ans. A

126. Last stage of puberty in girls

A. Puberche

B. Telarche

C. Menarche

Ans. C

127. A lady having symptoms of menopause


A. Increased FSH and LH

Ans. A

128. Pacinian corpuscles

A. Unmyelinated fibers

B. Unencapsulated

C. Detect low frequency vibrations

Ans. A

129. About right principal bronchus

A. More wider than left

B. Runs obliquely

C. More prone to obstruction

D. Longer than left

Ans. A

130. Tickle sensations

A. A alpha fibers

B. A delta fibers

B. A beta fibers

Ans. A
131. Pure water loss replaced by

A. Isotonic saline

B. Hyopotonic saline

C. Ringer lactate

D. Normal saline

E. 5% dextrose

Ans. E

132. Factor common in facilitated and co-transport.

A. Both for inorganic ions

B. Uphill

C. Require carriers for transport

Ans. C

133. A female with bleeding and coagulation defects. Intrauterine death has
occurred. Management?

A. Platelet transfusion

B. Delivery

C. Whole blood transfusion

D. Fluid replacement

Ans. B

134. Free radicles produced in reperfusion injury after MI by

A. Monocytes/macrophages

B. Neutrophils
Ans. B

135. Nucleus ambigous arises from

A. Pons

B. Mid-brain

C. Medulla

D. Cerebral cortex

Ans. C

136. A person having anti-sera A and D with agglutinins B. Blood group?

A. B-positive

B. A-positive

C. O-positive

D. AB-positive

E. B-negative

Ans.B

MARCH 2017 RADIOLOGY PAPER 1 AND 2

1.Pt.is living in desert are and use plenty of water.from which segment most of water absorb?

Pct

Dct

2.Which nerve , when destroyed bilayerally lead to death?

Fascial

Trigminal

Vagus.

3.Duodenum lymph drains into


Celiac n superior mysenteric

Superior mysenteric

Celiac

Inferior mysenteric

4.partial thirst IS aleviated by?

Git distension

SIADH

inc osmolarity

5.Female , hb 5.6 tlc 3500 platelate 1,20,000. Weakness during walk tingling in feet.what is cause?

Def.folic acid.

Vit.b12 def

Aplastic anemia.

6.Tymus arterial supply by

Internal thoracic n inf thyriod

7.alpha adrengic stimuliaon

brochodilation

vasodilation

.pupilary dilatation

8. Prostate arterial supply

Inferior vesical artery

9.Thyroid invested in

Pretracheal fascia

10Tracheal isthmus ..........c2_c4

11.Pt recive trauma head now pulsation with each respiratoon in eye.cause?
Cavernous sinus+internal carotid

Cavernous sinus+external carotid

12. In lactation no pregnancy occour why?

Releas of prolactin

I nhibition of gonodrotropins

13.Right sided paralysis n left sided facial deviation

Pons

Internal capsule

14.After c.section patient feel pain in lumbar region with urinary symptoms structure injured

Ureters

15.A police man took opiate n cannabean what finding u wil see in him?

Euphoria

Denial

Antisocial behaviour

16.Medial epicanthus ligament attach at?

Medial canthal

Nasal

Fronto nasal

Ant and posterior canthal

lateral and medial lip of lacrimal bone

17.Presence and absence of vomiting

Nominal

18.Selective age group selected then randomly picked sampling ka pocha that

Random startified

19.What is most true aboul parallel breast nodes?


Internal nodes to parasternal

Axillary node to axilla.

20. Inf.rectal drains in

Int.pudendal

Ext.pudedal

21.After surgery of bone regenration occour throuh?

Periosteum

22.Outer layer of nerve fiber regenrate at?

Epineurium

23.Which one will not be present in spinal cord at L4 level ?

lateral horn

Grey matter

Ant horn

Ventral horn

Dorsal horn

24.Complication of diabetics

Dry gangrene

Wet gangrene

25.Node of ranviar deficient at

Axoplasm

Myelin sheath

26.Which of following are present at most posterior of diaphragm?

Esophagus,aorta,thorasic duct

IVC,Aorta,thorasic duct

Left vagus,azygos,aorta
Right vagus, thorasic duct,aorta

aorta,azygous vein and thoracic duct.

27.peptic ulcers pain transmission

Greater splanic

Lesser splanic

Least splanic

28.Most of organs in embronic life laid down during which month

4th week

2nd( 8 weeks)

29.A man fall from height...Urinary prob..Sacral segment involve

S2,3,4

30. At 7_8 week mostly gonadal ridges of both sexrs same, but what point wil difrent in boys

Guberculm regression is must to descrnd tested

Regression og guberculum id called swelling method.

31.Heparin required which factor

AntiThromboxin 3

32.best receptor induces drug example?

Protamine antagonise heparion

Mannitol diuresis

Labetalol bradycardia

33.Cardiac plexsus contain

Both sympathetic and parasympathetic


34.Cimentidine n sucralfate can be given togather because

Cemtidine inhibit sucralfate metabolism n increases its toxicity

35.After birth what happens?

Truncus arteriosis become ligamentum arteriosis

Foramen close and Dec LV pressure

36.After surgery sinus and yellow granules?

Actinomycosis

37.Basillar artery divided into

Aica

Post cerebral

Sup cerebral

vertebral

38.bleeding from deep cervical....press costocervical to stop bloood

39.Pt has itching.Pt.cant bent his arm back to scrach his back due to lesion of?

Seratis anterior

Latismus dorsi

Teres minor

40.Aorta n pulmonary can't separate defects lies in

Truncus arterious

41.Erbs palsy lesion involve?

C5 and c6

42.Time taken by impluse to reach from endocardium to epicardium

Ventricular activation potential

Pr interval

RR interval
QT interval

St segment

43.In carpal tunnel syndrom due to ulnar nerve involvement which muscle involved?

Abductor policis

Flexor carpi ulnaris

Extensor carpi ulnaris

44.Defect in interventricular septum valve involve

Triscpid

45.Subacute endocarditis cause

Strep viridan

46.upination n extension is finction of?

Radial+musculocutaneus

Ulnar+musculocutanrious

Deltoid

47.Musle of lower trunk of brachial plexus?

Supraspinitus

Deltoid

Teres minor

48.Acrosomal cap formed by

Golgi appratus

49.Mediator of allergy?

Bradykinn

5ht

50.Duodenum 1st part arterial supply

Sup pancreatocoduodenal
INF pancreatocoduodenal

Right gastric

Right hepatic.

supraduedenal

51. Pyramidal corticospinal tract?

Dynamic redponse.

52.Head of pancreas ant to

Aorta.

Ivc.

Sma

53. Portal vein dilatation on abdomen with systemic circulation ?

Paraumbilical veins

54.Foreign body get obstructed in

Right segmental

Right main bronchus

Left inferior

Left apical

Left main

55. In which of gollowing vertebrae involved but NO spinal cord involved?

Cordoma

Anencephaly

spina bifida occluta

Omphelocel

Sacrococcygeal tumor.

56.surgically breast is divided into?


Lobule enlarge in puberty

Lobular stroma attached.

57.Portal vein formed by smv n splenic vein at lower border of

Duodenum

Transverse mesocolon

Spleen

Head of pancrease

neck of pancreas

58.True about fallopn tubes?

6cm

Laterally move to ovaries medially

Infudibulum has tortous structure

Ampulla have fimbria

59.Multiple nodule in liver on biopsy adenocarcinom what is tumor marker?

Cea

alpha feto protein

60.True about Esophagus?

Commence at cricoid

61.55 yr old male suffering from DM since 25 yrs...kidney issue?

Kimmel syndrom

62.Intestinal tumor resected in a child with thousands of polyps

Father died with same disease.Histology show

Adenomatus

Tubular

63.Stagnant hypoxia?
Polycythemia

Cynide poisning

Co monxide poisning

Severe anemia

64.common hepatic duct Cholangiography scan , branch after hepatic duct winds around a target
organ .?

Cystic duct

Cbd

Hepatic duct

65.A lady in kitchen cut her radial artry accidently which mechanism wil reduce presure in vessel and
stop bleeding?

Adelta fiber

Beta fiber

B fiber

A or b1 fiber

C fiber

66.During accudent a lordotic thin man got 3 cm wound near umblicus on right side.

Initially vitals were normal but aftet sometime bp low..which structure involved?

Ivc

67.midbrain lesion involve nerve?

Occulomotor

Vagus

Trigemminal

Vagus

68.Pt had m.i

Inferior wall infarct on ecg..artry involved?


Rt.marginal

69.patient have hearing prob,he can't see laterally,have fixed pupils ?

Cerebrpontine angle

Mid brain

70.Atrioventricular node(A.V node) supplied by?

RCA at origin of aorta

Left coronary artry

RCA because it is present in posterior interventricular septum

71. Septomarginal (Moderator band) present in?

Rt ventricle

72.Pnemothorax scenario for cvp line vessele used?

Left juglar

Rt juglar

Subclavian

73.A female presents with Arterial disease on examination u find pulsatile mass on abdomen it
originates from

T9-T11

T12-L2

S2-4

S5-4

L3-l5

74.Pnemothorax start at lower limb 1st at?

Pilmonary artry

75.Pnuemothorax?

Chest wall wil collapse and spring out

76.Duedenal perforation artry involve...gastroduedenal


77. Female got stab wound in chest...Hyperresonant node,absent sound on right... Mediastinum
towards left

Right pneumothorax

Hemothotax

78.Umblical pain dermatome?

T10

79.inestinal pain shif to?

umblicus

80.Femoral artery pulsation feel at

Mid inguinal point...

Mid point at ingunial ligament

81.Boy had high grade fever since 4 days .took anti malerials now cola color urine...g6PD

82.A pt has inc glucose but after taking fructose glucose reading normal which is involved?

Glucose6 phosphate

Tyrokinasa

83.Hepatodudenal ligament contents

Common bile duct,portal vein,hepatic artery

84.Portal hypertension vein involve

INF epigastric

Esophageal

Imv

Smv

left epigastric

85.Pt is working in smokeless cigarette factory he has a nodule on biopsy sub mucous fibrosis what
could be lesion?

1-dysplasia 2-metaplasia 3-precancerous lesion


86. A pt has white thick patch in mouth which cnt be removed by pressure with hand what it can be??

Leukoplakia

candida

87.Cervical cancer initial lymph metasis to?

Internal illiac nodes

88.A woman has cervix biopsy with large nuclei she came again after 6 months for follow up lesion?

Dysplasia

Metaplasia

Hsv

89.Gastric emptying increase by?

Gastrin

Cck

90.Antipyrine n heavy water use to diagnose

Total body water

Ecf

Icf

91.To increase ECF volume...

Hypertonic saline solution

92. True about CSF?

Protein more than plasma

Pressure more than dura

Absorb by choroid

Sometime enter in lymph.

slightly acidic than blood.

93. 9% N/S solution


9gm in 1000ml of distilled water

9gm in 100 ml distilled water

9mg in 100 ml distilled water

94.Regarding inter costal collateral vein in chest wall?

2 in mumber

Supply chest muscles

Give rise lateral cutsneous branches

Present in subcostal groove

95.suprapubic catherization done from the surface not cover by peritoneum ?

Anterior surface is not coverd by peritonium

96.Chelosis dermatitis vesicles deficiency of.?

Niacin

riboflavin

97.Wound strength provided by

Type 1 collagen

Type 2

Type 3

98.Base of bladder oblique ridges called?

Rice bar

Merces bar

intruretic bar

99.Lateral fornix of vagina is closely related to

Uterine artery

Uterine tube

Ureters
Round ligament

100.uncinate process of head of panreas has a ridge for?.

SMA

IVC

AORTA

101.During football game a boy injured and now unable to dorsiflex foot..dorsal sendation is
normal...nerve involved?

common peroneal nerve

102.A boy injured neck of fibula now asking for artry?

anterir tibial

103.During game boy injured which area of eyeball that now there is vertical diplopia and
enophthalmos?

Apex

Base

Roof

Floor

104. pt has joint pain investigation showed HLA DR4 Association

RA

105.Vertcal movement of thorasic cage.?

Contraction of diaphragm

Bucket handle

106.Female having hb5 GGT raised...Hepatomegaly,pruitis or bhe bht symptoms thy test would be

Antimidrocondrial antibody

107.A dietician doctor/nutritonis will advice a pt or client that contain more vitamins?

Egg

Milk
Green vegetbles.

108.prutits with deranged lfts specially GGT 200

Extra hepatic cholelithiasis

Intrahepatic

Gall stones

109.Metasis occurs due to

Loss of E-cadherin

110.A pregnant lady took only ferrous sulfate in her diet which will b deficient in her?.

Vit.d

Vit b

Folic acid

111.most disabling feature of cerebellar lesion?

nystagmus

112.Pregnant lady deranged lfts go to remote area ?

Hep E

113.25 year para 4 has occlut blood in stool?

Iron deficiency anemia

114.Suprabuic inscion

Bleeding is due to

Sup epigastric

INF epigastric

Deep circumflex

115.Young boy with inflamed big toe febrile, raised TLC ,UTI,urea level raised.For Diagnosis?

Monosodium urate crystal in synovial fluid

Synovial fluid for culture and sensitivity


116.Cruciate ligament of knee joints

Intracapsular

Surrounded by synovial membrane

117. female has Dull,steady,epigastric slightly left sided pain radiating to back is

Duodenum

Pancrease

Gall bladder

Stomach

118.Heat lose via endotracheal tube by

Evaporation

119.In all patients of anthrax 90 % the lesions of organ must invlove ?

Skin

Heart

Lungs

120. true hermaphrodite 46 xx . ambiguous genitals

cause?

adrenal hyperplasia

CAH

121.pulse pressure increase due to?

Cardiac output

Stroke volume

EDV

TPR

systemic?

122.basophillia in cell is due


GOLGI

SMOOTH ENDOPLASIMIC RETULUM

RIBOSOMES

rer

123.calcium inside skeletal muscle sarcoplasm attaches to

calmodulin

124.mother blood group is A 2 children with O and AB father blood group?

125.when drug is bound to albumin

it is excreted through kidney

Inactuve until activatd n liver

temporary inactive

126.which of followong acts by cAMP

TSH

ADH

127.Oat cell CA tumor markers

ACTH.

128.head of pancreas posterior to..... SMA

129.Hb 5,direct bilirubin inc retic 10, ALP and ggt normal.

Hemolytic anemia,

stone in bile duct

130.patient with parotid gland Swelling. stones in found in duct. which proceess is inbolves

apoptosis

necrosis

131. Secretions from zona gromelulosa depends on


Angiotensin 11

ADH

Renin

Potassium

ACTH

132.A student witnessed 1st time a surgery got collapsed with decrease BP...Cause?

inc TPR

Increased cardiac output

Decrease cardiac output

decrease TPR

133. patient wid hypotention , CO is 2.lit / min. jvp is 2.mm mercury f water. lactate raised cause ?

hypovolemic shock

cardiac temponade

congestive cardiac failure.

septic shock

134. smooth endoplasmic reticulm abundant in

nerve cells

cardiac cells

skeletal cells

gonadal cells

exocrine gland and leucocyte

135.Insulin independent transport of GLUCOSE in?

exercising Skeletal muscle

Heart

Kidney
Thalamus

Resting skeletal muscle

136. organ which is completely dependent on glucose for energy?

heart

liver

brain

skeletal.muscle

hypothalamus

137.Q: 3 months baby presented wd critically ill condition..PH:7.2

PCO2:32

HCO3:14

Na: 142

K:2.5.Diagnosis???

a.Addison disease

b.compensated resp acidosis

c.diabetic acidosis

d.lactic acidosis

e.renal tubular Acidosis

138.?nucleus disappear in which stage ?

pronormoblast

reticulocytes

normoblast

139.chronic asthemetic now aggravated symptoms.

ph 7.2 bicarb 24 ?

resp acidosis
resp alkalosis

uncomponsated metabolic acidosis

140. Circulation doesn't compromise during moderate exercise

Coronary

Cerebral

Renal

SK muscle

141.Na/k atpase pump is stimulated by?

Adrenaline

Noradrenaline

Insuline

142.Sever diarrhoea ?

normal anion anion gap metabolic acidisis

143. Infective endocarditis scenario. Damage ?

Damage= thrombus= bacterimia= perforation

144.Sickness is defined as

A.What physician thinks

B.illness that patient feels

C.whats the clinical diagnosis

D.whats the lab suggest

145.Most common feature of renal tubular acidosis

A.hyperkalemia

B. Proteinuria

C. Increase urea n normal creatinine

146.regarding basal ganglia?


Intrnl capsule medial to GP

Caudate forms neostriatum

internal capsule lies medial to red nucleus

147.Pyramid corticospinal tract?

contain cortical neurons.

Axons arise from parietal and pre motor

Projecting fibers

decursate at corpus callosum.

olivary

148.2nd ordr neurons of nociceptrs are locatd in?

post root ganglion.

post horn

Ventral root ganglion

Ventral horn

dorsal root ganglion

149.a man had injury after that presents with loss of 2 point discrimation at T8 on same side..N pain
loss on contralateral side lesion would be at.?

A spinothalamic tract at T8?

Hemi section of cord at T12

Springomyila.

T11?

150.Warty growth ?

Condyloma accumnata

151 A boy presented with hemorrhoids doc examined INF rectal vein...That drains into ?

Internal illiac vein

Internal pudendal
External pudendal

152.Appendicular artery is a branch of

Middle colic

SMA

Ilioceacal

post cecal

153.a baby came with intestinal obstruction hushrprung disease. involve parasympatheticsegment ?

S2,3,4

S4,5

L2,3

L4,5

154.Cells that transform into mast cells n release histamine ?

Monocytes

Neutrophils

Macrophages

basophils

esionophils

155.Erythroid stem cell generates in bone marrow under the influence of

Thyroxine

Growth hormone

Growth factor

Erythropoietin

Cortisol

156.st defence of body against bacterial infection

Complement system.
Skin

Opsonization

Chemotactic

No relase

157. in injured tissue Chemotatic response by?

C3a

C5-9

InterlukeinsB4

TNF

158.Pencillin rection main feature?.

fever

anaphlaxis

joint pain

bronchospasm

Rash

159.Sesamoid cartilage present in

Ala of nose

Layrnx

Biceps femoris

160.CA of colon metasis 1st to

Liver

Kidney

Lymph nodes

heart

161.Femoral artery relation?


Medial to femoral nerve

Lies in medial compartment

Lies Infront of inguinal ligament

Enters thigh by pass through adductor canal

pass between adductor longus

162.Superior thyroid artery is related to

ELN

RLN

163.Lymph drainage of glans n corpora

Sup inguinal

Deep inguinal

Paraortic

164.s4 heart sound produced by?

Rapid ventricular filling

atrial systol

165. ductus deferens terminates in ?

seminal vesicle

ejaculatory duct

epidiymis

167.M.c Reason of tuberculin test negative in tb ?

pt= taking immunosuppressant

168.In an adult the common site of urinary stone obstruction

Uretropelvic junction

Where it enters bladder

Where it crosses internal illiac


pelvic brim

169.primary syphilis dx can be made by scraping ?

mouth ulcer

genital lesion

skin

170.Commonest presentation of vitamin a deficiency ?

Night blindness

171.appendix supplied by....SMA

172.Cervical branch of facial nerve supplies

Playtsma

173.Bulbuar urethra rupture urine extravagate into

Superficial perineal pouch

Deep perineal

174.During episotomy muscle cut

Bulbospongious

Levator anni

Perneal membrane

175.most common cause of cancer

over expression of protooncogenes

176.a child is continuosly breathing in a plastic bag his resp rate has increased best option

increase in alveolar PCO2

increase in arterial PCO2

increase in PO2

177. for aretrial blood gass sample taken?

Whole venous blood


Whole arterial blood

Centrifuged plasma

hepranized

citrate

178.( apex)zone one of lungs

Ventilation high

Perfusion high

V/q increases

dead space high

shunting high

179.Thyroid gland originate from

Foregut ventral side

Foregut medial side

Endoderm

Mesoderm

4th pouch

180.Pt has extarnal oss in introitus and heaviness in perinium.? what is damage?

uterosacral ligament,

levayor ani and perineal body

181.Fibrocartilage?

Amorphous material

Present in disc.

182. Plasma and interstitial osmolarity diff?

1mosm

183.Dipthrria toxoid hypersensitivity reaction?


type 3

184.Mentally normal height less, achonrdoplasia?

auto dominant

185.Which one is diffrent from others?

Osteoclast

Osteoblast

osteocytes

chondroblast

186.Collecting duct?

Projection fibers

Interlobar

medulary rays

187.On autopsy in crf enlarged structure found is

Parathyroid gland

188.about pseudostratified?

not present in lungs

all Have tall height

all Lies nuclei at same level at botom

189.HELP syndrom?

hemolysis,elevated liver enzymes, elevated platelates

190.A 14 years old boy with 8% creatinine bp 180/110 stucture involved is

Jg cells

Glomerulus

191.UV prolapsed most common complication?

Discharge
Chronic cervicitis and metaplsia

192.Chromosomal maneuring doesnot occurs in..?

asexual reproduction

cleavage.

meiosis 1..

miosis-2

193.True about RBC?

Covex

Carry gases

Mantains PH

194.Corpora is injured why should it repair?

For erection

1st MARCH 2017 OPHTHALMOLOGY PAPER (Compiled by Dr.


Osama Javaid)

Note :
- Below mentioned stems/options/lines/answers were taken
from a post that we made specifically for recalling the paper
questions and their answers on the fb group "friends who
like fcps part 1 ophthalmology" after the paper.
- Complete questions and all the options could not be
recalled. So if only two three options are written that's
because only two three options were recalled.
- We are writing below what members could recall and the
options they were mostly agreed upon.
- All questions are mixed from paper 1 and 2 though I tried to
keep the eye related questions at the end and general MCQs
at the start.
- During compiling, I got a slight suspicion that few MCQs
were not present in everyone's paper, means some questions
were different for everyone (I don't know if this suspicion is
true or not but may be it's a real thing and CPSP se kuch b
tawaqqo ki ja skti hai)
- IF YOU CAN RECALL/COMPLETE/CORRECT anything, you are
more than welcome to do so.

1- A boy fell from building, urinary incontinence, nerve loss =


S2 to S4

2- Initial sign of acute tubular necrosis = concentrated urine


3- 4th heart sound is produced by = Ventricular wall vibration
due to atrial kick/atrial systole

4- Aortic insuffeciency will happen in = options were SLE,


Scleroderma, Dermatomyositis, RA

5- Which is a chemotactic agent = options were C3b, LTB4

6- For taking vitamins, what should be advised to take in diet


= green vegetables

7- Atropine is given before surgery for = Decreasing


secretions of bronchial tree

8- Which drug has action same as Atropine = Scopolamine


(Rabia Ali MCQ)

9- A question was that which of the following hepatitis has P-


ANCA ? = Options were A, B, C, D and E
10- HLA-DR4 is diagnostic of = Rheumatoid Arthritis

11- A question regarding that which of the following disease


incidence increased in last 5 years in Pakistan = Options mein
leprosy, polio, hepatitis they, I marked hepatitis but i am not
sure.

12- How a surgeon can save himself from hepatitis B infection


= Many candidates marked 'to avoid the use of sharp
instruments', another recalled option was to screen the
patients but I guess this is not true because screening only
shows whether someone is Hep B positive or not and
surgeries might also have to be performed on Hep B positive
patients so in my opinion the option is to avoid the use of
sharp instruments.

13- Heart act as a syncitium because its cells have = Gap


junctions

14- 1st step in thyroid hormone formation = Binding of Iodine


to Tyrosine (Rabia Ali MCQ)
15- A question was regarding when ligating superior thyroid
artery which nerve could get damaged = External laryngeal
nerve (if asked inferior thyroid artery then answer would be
recurrent laryngeal nerve)

16- Which of the following condition of the thyroid most likely


lead to malignancy of thyroid = Non-functioning nodule

17- A scenario about thyroid related bilateral proptosis and


they asked which of the follwing investigation is best for
diagnosing it = Options were T3/T4 levels, TSH, Thyroid scan,
antithyroglobulin antibody (I marked TSH)

18- Thyroid enclosed in a condensed sheath = Pretracheal


fascia

19- Severe diarrhea will cause = People marked Metabolic


acidosis with normal anion gap (I marked with increased
anion gap which is likely wrong)

20- All important cerebral vessels are in = subarachnoid space


21- Diagnosis of syphilis by = Genital sore scrapping

22- HIV causes = Decrease in CD4 cell count

23- Feature of well differentiated squamous cell carcinoma =


Presence of nest of keratin

24- First line of defense against microbes = We marked skin


('first' line of defense is skin, then comes macrophages
neutrophils etc, other options were complement, opsonin)

25- An extended JVP, low B.P, high lactate scenario = Cardiac


Temponade

26- Tyrosine kinase receptors are present in = Cell membrane,


Cytoplasm (I marked cell membrane)

27- Which hormone act by c-GMP mechanism =


28- A question regarding superficial cardiac plexus = We
marked 'it contains both sympathetic and parasympathetic
nerves'

29- Which will increase ECF = infusion of hypertonic saline

30- Which will cause urine osmolarity to increase when a


person drinks less water = ADH

31- Posterior cerebral artery is a branch of = Some marked


basilar artery

32- Heparin and Histamine are released from = Basophils

33- Normal saline composition = 0.9mg NaCl in 100 ml of


water (I marked it, other options were 0.9g in 100ml, 0.9mg
in 1000ml, 0.18mg in 2000ml and so on)

34- Thyroid isthmus is at the level of = 2nd to 4th tracheal


rings
35- Example of sesamoid cartilage = options were epiglottis,
larynx, ala of nose, pinna of ear, external auditory meatus (I
marked epiglottis but I am not sure)

36- A question regarding blood supply of thymus = We


marked inferior thyroid artery + internal thoracic artery
(other options were Costocervical, Thyrocervical,
Suprascapular)

37- A baby developed jaundice on third day of birth presented


on 2nd day (WTF CPSP?) , Which is the investigation of choice
? = Unconjugated bilirubin (According to a candidate it was
the scenario of physiological jaundice)

38- A patient had jaundice for two months and itching for two
years, Diagnosis = Options were extrahepatic cholestasis,
intrahepatic cholestasis and few others (I marked
extrahepatic cholestasis)
39- Patient presented with warty growth on vulva, Likely
diagnosis = options were Squamous papilloma, squamous cell
carcinoma, condyloma acuminatum

40- Which of the following will increase in pulmonary


embolism = options were Arterial pO2, Arterial pCO2, Alveolar
pO2, V/Q ratio

41- What change will occur in uterine prolapse = options were


metaplasia of cervix, dysplasia of cervix, chronic cervicitis (I
marked dysplasia)

42- Grading of cancer can be done by = options were No


of.cells, mitosis, Type of cells, Permeability of membranes
(Not sure whether the true answer is within these recalled
options or not)

43- Which tumor never metastasize = Basal cell carcinoma

44- Submucosal fibrosis is = Premalignant change (According


to many candidates this is the true answer, I also marked this)
45- Which of the following nerve if cut bilaterally will result in
death = We marked Vagus Nerve (Options were CN5, CN7 etc)

46- Which is the important factor in tumor metastasis = Loss


of E-Cadherin

47- A question regarding internal carotid artery = Correct


option was that it is lateral to external carotid artery at the
bifurcation of common carotid and then passes posteriorly
and turns medially

48- Acrosome formation occurs in = Options were lysosome,


golgi apparatus

49- Basophilia in pancreatic cells is due to = Ribosomes

50- Which of the following releases in allergy = I marked


bradykining but I guess this option is wrong
51- Regarding basal ganglia = internal capsule is medial to
globus pallidus (Rabia Ali MCQ)

52- Sickness is = feeling of being unwell perceived by the


patient (Candidates marked this, other options were like
labelling of disease, diagnosis of disease by tests etc)

53- Mother is blood group A and children have blood groups


O and AB, What is the blood group of father = I marked B

54- Which is the first pre-requisite for a graft = ABO


compatibility (see 'first' in the stem, otherwise there are
many other things which have to be considered in grafting)

55- Which skin malignancy is commonly associated with HIV =


Kaposi's Sarcoma (Rabia Ali MCQ)

56- Haemoglobin binds to = Heptoglobin (I think it was asked


which of the following is decreased in hemolytic anemia and
the answer was heptoglobin)
57- In tracheostomy heat is lost through = Evaporation

58- A patient having chelosis and angular stomatitis having


deficiency of = Vitamin C

59- Psychoactive substance related with memory = options


were cocaine, alcohol, amphetamine, caffeine (Candidates
marked caffeine)

60- A policeman presents with history of substance abuse,


what will be the finding = options were denial, antisocial
behaviour, grandeur etc

61- Regarding CSF = options were more acidic than plasma,


more proteins than plasma, dural sinuses has less pressure
than ventricles (I guess this last one was correct option)

62- Scenario of baby born with swelling on the back with


neural tissue in it and raised intracranial pressure =
myelomeningocele
63- A bone is surgically removed, it will be replaced by =
options were compact bone, spongy bone, periostium

64- Outer layer of a nerve fibre is = epineurium

65- A boy in a party started breathing in a plastic bag, what


will increase = options were arterial pco2, arterial o2, alveolar
pco2, alveolar o2

66- Which cell has a life history different than others =


options were chondroblast, chondroclast, osteoblast,
osteocyte, osteoclast (Few candidates marked osteocyte,
Few marked osteoclast)

67- Mechanism of action of acetylcholinestrase = Increase


acetylcholine at motor end plater

68- Renal columns contain = options were interlobar arteries,


interlobular arteries, arcuate arteries, calyces
69- A man in the desert has a habit of drinking plenty of
water, where is maximum percentage of the water is
reabsorbed = options were PCT, DCT, loop of Henle,
collecting tubule (Answer was PCT because no matter what is
the condition, maximum percentage is reabsorbed from PCT)

70- Which of the following increases the activity of Na-K


ATPase pump = options were Adrenaline, Insulin (some
marked adrenaline, some marked insulin (I also marked
Insulin)

71- Which antibiotic act on neuromuscular transmission =


options were gentamicin, penicillin, chloramphenicol,
metronidazole, ceftazidime

72- Adverse effect of isoniazid can be prevented by taking =


Pyridoxine (Rabia Ali MCQ)

73- Antiobiotic having decreased clearance in chronic kidney


disease = Gentamicin
74- Drug bound to plasma proteins = options were
temporarily inactive, temporarily inactive unless activated in
liver, gives effect in blood only, can cross blood brain barrier

75- Neurogenic shock commonly presents with = options


were increased cardiac output, increased pulse rate,
decreased peripheral resistance, decreased vasomotor tone,
cold hands (I marked decreased vasomotor tone)

76- A scenario regarding dividing the population in groups on


age basis and then taking random samples from each group =
Stratified random sampling (Rabia Ali MCQ)

77- Insulin dependent (some candidates were pretty sure that


the stem contained the word INDEPENDENT) glucose uptake
occurs in = options were exercising skeletal muscles, resting
skeletal muscles, adipocytes, brain cells

78- In infective endocarditis, the sequence of events is =


Damaged valve, Thrombus formation, Bactremia, Prolapse
(Given in Tanveer's Notes as the pnemonic 'DTBP')
79- High cardiac output in = Beri Beri

80- In I.V drug abusers = Infective endocarditis

81- Subacute bacterial endocarditis, organism responsible is =


Streptococcus Viridans (Rabia Ali MCQ)

82- In anthrax = skin lesions are most likely

83- Multiple lung abcesses, organism = options were


Stapylococcus (I marked this), Pneumococcus, Klebsiella

84- Impulse travelling from endocardium to pericardium is


represented on ECG by = options were QRS, QT, RR, PR (I
marked QT but I am not sure)

85- Impulse directly transfers to ventricles by = options were


AV node, bundle of his, right and left bundle branches,
purkenjie fibres (Many candidates marked purkenjie fibres)
86- A female is pregnant and she is taking ferrous
supplement, what deficiency she is prone to develop = Many
people marked folic acid, another option was Vit D

87- Descending tract related functionally with = options were


corticospinal tract, vestibulospinal, reticulospinal

88- L4 level spinal cord does not contain which = options


were White matter, Grey matter, Lateral horn, Ventral horn

89- Thrombus in deep vein of lower leg will go into = We


marked pulmonary artery

90- ABGs sample is taken from = From artery in a heparinized


vial (This option was marked by majority)

91- A scenario of acidosis

92- Stagnany hypoxia occurs in = options were COPD,


Asthma, CO poisoning, Polycythemia, Cyanide poisoning,
Severe anemia
93- Heavy water and antipyrine are used to measure =
options were ECF, ICF, Total Body Water (I marked)

94- Intracellular lipid peroxidation is indicated by = Lipofuscin

95- Systemic amyloidosis scenario

96- Male with short limbs, normal trunk, lumbar lordosis,


Condition is = options were autosomal dominant, autosomal
recessive, X-linked dominant etc (People marked it as
Autosomal dominant because of diagnosis of achodroplasia)

97- Presence or absence of vomiting is which type of data =


Nominal (Nominal is the type of data with only two options
e.g, yes or no, presence or absence etc)

98- Penicillin hypersensitivity reaction feature = Many people


marked Bronchospasm (I marked skin rash which is likely
wrong)
99- Complication of diabetes = Dry gangrene

100- Increase pulse pressure is due to = Increased stroke


volume

101- Organ having maximum difference of oxygen in arteries


and veins = options were liver, heart, skeletal muscles (Many
people including me marked heart, this is basically the
question of organ using maximum oxygen at rest, they
changed the wordings of stem)

102- Organ needing continuous supply of oxygen = Brain

103- Which organ's blood flow doesn't change significantly by


moderate exercise = Brain (may be the stem have moderate
haemmorhage instead of moderate exercise but I am not
sure)

104- Foreign body will most likely lodge into = Right main
bronchus (I didn't mark right segmental bronchus because I
thought that segmental bronchus could be superior too
instead of inferior so I only marked right main bronchus)

105- Tuberculin test is negative in a known TB patient, Reason


= Taking immunosuppressants

106- Cervical branch of facial nerve supply = People marked


Platysma

107- Cause of cancer = over expression of proto-onco genes


(few other options were Non lethal injury to gene, Absence
of one chromosomal gene)

108- A patient's BP drop after surgery due to = Decreased TPR


(I think the stem was something else)

109- A scenario of fibrocartilage = People marked Amorphous


matrix material (another option was 'present in disc in
synovial joints', this option is likely wrong)
110- Difference of intracellular and extracellular = 1 mOsm (I
am not sure about the stem though)

111- Monosodium urate crystals in big toe in gout (I don't


remeber this MCQ, one candidate written it)

112- Hyperurecemia is the feature of = Thiazide diuretics

113- Thirst is partially relieved by = Gastric distension

114- In pneumothorax = ipsilateral lung collapse and chest


wall sprung out

115- A scenario of drug interaction = Many people marked


heparin - protamin sulphate

116- In pseudostratified epithelium = all cells lie on basement


membrane
117- Difference between smooth and cardiac muscles = Ca-
calmodulin

118- A scenario of primary biliary cirrhosis, investigation =


Antimicrosomial antibodies

119- Asterognosia scenario

120- Biceps flexion = c5, c6

121- RBCs main function = exchange of gases

122- RBCs lose their nucleus in = Late normoblast stage

123- Most potent antiemetic = ondansetron

124- Chromosome doesn't replicate in = A candidate marked


asexual reproduction
125- Diptheria scenario = Type 3 hypersensitivity

126- Cotton mill worker will have = Byssinosis

-------------------------------------------------

EYE RELATED MCQs

127- An option of a question was conversion of cisretinene to


transretinene (I marked it but I don't know what was the
stem)

128- Nerve which passes through tendonus ring and superior


orbital fissure is = Abducent nerve

129- Iris dilator/sphincter muscles derived from =


Neuroectoderm

130- Cornea is kept moistened through = Greater Petrosal


Nerve
131- Function of lipid layer of tearfilm = To lubricate the eye
(They try to confuse you with one of the options 'to keep the
overlying aqueous layer from evaporating', so remember that
aqueous layer is UNDERLYING, not OVERLYING the lipid layer.

132- A question regerding the walls of the orbit and I guess


the correct option was that floor of the orbit forms from
Maxilla and Zygomatic bones

133- There was a question jiski correct option ye thi k greater


wing of sphenoid is a part of lateral wall of the orbit.

134- In Fuch's dystrophy = Loss of endothelial cells of cornea

135- Type of cataract formed after trauma to lens is = Rosette


shaped

136- A question was about pathway for corneal source of


energy, options mein there were glycolysis, pentose
phosphate pathway etc, Many members marked pentose
phosphate but we are not sure.

137- What kind of cataract forms in juvenile diabetes =


Snowflake Cataract

138- A question was like 'when a person looks near, what will
decrease?' = I marked 'area of the retina excited by light
(because I thought that on looking close, the pupil gets
miosed and the are of the retina getting the light decreases),
one candidate marked visual field

139- Alpha receptor activation/agonists will cause = Mydriasis

140- Nerve supply of levator palpebrae superioris = Superior


division of oculomotor nerve

141- Grey line at the end of the eyelid is formed by = Options


were mucocutenous junction (which I marked), end of
orbicularis oculi, End of tarsal. (I remember that I read in a
past paper that grey line is formed by tarsal plate but when I
found no such option then I marked mucocutenous junction.
Some other members also agree to this answer.

142- A scenario regarding that a patient can't see laterally


with his left eye and he don't feel sensations in the lower
right body, where is the lesion = Options were left internal
capsule (which I and one other member marked), right
internal capsule and right cerebreal cortex, can't recall any
more options.

143- A scenario regarding that a person can't close his eye,


which is damaged = Facial nerve (because it supplies
orbicularis oculi which closes the eye, if they ask that the
person can't OPEN the eye then the option would be
oculomotor nerve because eye opening is the funcion of
levator palpebrae superioris which is controlled by
oculomotor nerve)

144- A question regarding which of the following feature is


correct regarding cornea = Many people marked
Unmyelinated nerve fibres in the cornea
145- Scanning/Horizontal eye movements are controlled by =
Frontal eye fields

146- A scenario regarding bulbar conjuctiva = Attached to


Tenon's capsule at limbus

147- A question about the width of surgical limbus = Options


were 1mm, 1.5mm, 2mm, 2.5mm, 1-3mm (I marked 2mm but I
am not sure)

148- A scenario that when a person slowly turns his head to


the right then what will happen = I marked that the
endolymph in the both semicircular canals move in opposite
directions but there were few other options too like
kinocilium hair movements but I can't recall and i am not sure
about it either.

149- There was a question about the Bowman's layer of


cornea = options were Continous with trabecular, endo can
regenrate in kids, growth complete in 3 years but i think none
of them is true option.
150- Optic canal opens in = Middle cranial fossa

151- Lens thinnest at = Posterior capsule

152- Aqueous humor has which more than plasma =


Ascorbate / Ascorbic Acid / Vitamin C (All are the names of
same things, they gave Ascorbate in the options so we
marked it)

153- In which condition retinal degeneration occurs =


seperation of retinal pigment epithelium from neurosensory
retina

154- Krukenberg Spindle = options were visible corneal nerve


loops, 5 times more common in females, seen in pigment
dispersion syndrome (2 candidates marked it pigment
dispersion syndrome but I am not sure because some also
marked it as visible corneal nerve loops)

155- Choroidal capillaries are most fenestrated at =


submacular area
156- What to be given if patient develops bradycardia during
eye surgery = Atropine (It was indirectly the question of
oculocardiac reflex)

157- When light strikes the retina what will happen =

158- A question regarding RPE = Correct option was that it


covers the outer surfaces of rods and cones

159- Corneal reflex pathway goes to = I marked 'through


trigeminal ganglion to sensory cortex'

160- Optic vesicle arise from = Forebrain

161- Wilson disease scenario = Serum ceruloplasmin is used


for diagnosis

162- A tumor in optic canal will likely compromise which


structure = We marked ophthalmic artery
163- Homonymous hemianopia, where is the lesion = Optic
Tract

164- Fibres from LGB to visual cortex travel through =


Geniculocalcarine tract

165- Upper and lower eyelids lateral portions drain into =


Preauricular lymph nodes (Many candidates marked this)

166- A scenario regarding a child having intermittent


nystagmus which is irregular and on examination ophthalmic
and neurological examination is normal, where is the lesion =
Many marked cerebellum but I am not sure. I also marked
cerebellum

167- Vitamin A deficiency sign = Night blindness

168- What will happen in facial palsy = Epiphora due to lower


lid ectropion/sagging (one of the options were keratitis and
conjuctivitis but i didn't mark it because in my opinion
keratitis will happen when there is no tear production, in this
case tears are present but they will flow on the face due to
sagging to lower eyelid/ectropion but few candidates think
that due to facial nerve palsy the orbicularis oculi will not be
able to close the eye which will lead to keratitis and
conjuctivitis)

169- If focal length = 0.75m, refractive power will be = 1.33


(because, power = 1/focal length in meters)

170- Regarding visual cortex = options were Peripheral retinal


fibers end in upper calc sulcus, Macula has large posterior
location on visual cortex

171- Patient with history of meningitis 1 month back now


presented with bilateral papilladema and morning headache
= Hydrocephalus

172- Patient on ATT can't recognize between red green colors,


which drug is responsible = Ethambutol (Because it causes
optic neuritis)
173- A question regarding MLF lesion

174- Drug given in Herpes Simples Keratitis = Trifluridine

175- A scenario regarding ipsilateral 6th, 7th and 8th cranial


nerve lesion = Cerebellopontine angle injury

You might also like